首页 > 考试报名 > 初一英语考试试卷

初一英语考试试卷

发布时间:

初一英语考试卷

英语期末考试与七年级学生的学习是息息相关的。我整理了关于 七年级英语 期末考试题,希望对大家有帮助!七年级英语期末试题 Ⅰ. 听小对话, 选择图片(共5小题;每小题1分,满分5分) 1. What does Bill have? A. B. C. 2. Which is the girl’s family photo? A. B. C. 3. What’s Bob’s favorite thing? A. B. C. 4. What does the boy’s uncle want to be? A. B. C. 5. What animal does the girl like? A. B. C. Ⅱ. 听小对话,回答问题(共5小题; 每小题1分,满分5分) 6. Who has the ping-pong bat? A. Lucy B. John C. Peter 7. What number is the woman calling now ? A. 7045386 B. 7043586. C. 7045368. 8. How much is the T-shirt? A. 18. B. 16. C. 80. 9. What color does the man dislike? A. purple B. brown C. black 10. Where does the conversation happen? A. Classroom. B. Bookstore. C. Park. Ⅲ. 听长对话,回答问题(共5小题; 每小题1分,满分5分) 听下面一段较长的对话,回答第11至12两小题。 11. What is the family name of the man? A. David B. Green C. Brown 12. Does the woman have an e-mail address? A. Yes, she does. B. No, she doesn’t. C. we don’t know. 听下面一段较长的对话,回答第13至15三小题。 13. Where is the big box? A. Under the table. B. On the chair. C. On the table. 14. How old is the boy? A. Five. B. Six. C. Seven. 15. What’s in the big box? A. A toy car. model plane plant Ⅳ. 听短文,回答问题(共5小题; 每小题1 分,满分5 分) 根据短文内容,选择最佳答案。 16. Are Tina and Lisa students? A. Yes, they are. B. No, they aren’t. C. We don’t know. 17. They go shopping_____________. A. on Monday B. on Sunday C. on Saturday 18. What does Tina buy? A. A pen and some vegetables. B. A pen and some fruit. C. A pencil case and some fruit. 19. Who does Lisa buy some tomatoes and carrots for? A. Her mother. B. Her father. C. Her sister. 20. Where do they go after they buy the things? A. They go to school. B. They stay at the store. C. They go home. Ⅴ. 单项填空(共15小题;每小题1分,满分15分) 从A、B、C、D四个选项中选出可以填入空白处的最佳选项。 2l. Can you find the sound /æ/ in the following words? A. April B. box C. fat D. name 22. — What color is it? — It’s _______ orange. It’s _______ orange pen. A. a, an B. an, an C. /, an D. an, a 23. My mother is Alice Johnson. ________ telephone number is 2546809. 24. Are ________ Kate’s ________? , book ,book ,book D. these,books 25. — Is your photo in the drawer? —________. , I am ,it is ,it is ,I’m not 26. Let’s ________ and ________ football on the playground. go, to play , play go, play , to play 27. ________ your mother ________ carrots? ; eat ; eat ; eat D. Does; eat have sweaters ________ all colors ________ ¥50 each. A. in, for , in , at , at can ________ some food________ the supermarket. , to , from C. sell, from D. buy, to 30. Can you ________ it ________ English? , in , with , with D. say, in 31. Our school has a school trip ________ October 15th. 32. Mr. Smith teaches ________ science. We all like ________classes very much. A. us, his B. our, him C. our, him D. us, him 33. — ? —It’s Wednesday. A. What’s the day B. What’s the time C. What day is it today? D. What’s the date today? 34. — What do you think of the movie “Lost In Thailand”? —It’s . It makes everyone happy. A. busy B. difficult C. boring D. funny 35. — You look nice in this red skirt. —______. A. That’s nice of you. B. Oh, no. C. You’re welcome. D. Thank you. Ⅵ. 完形填空(共10小题;每小题1分,满分10分) 阅读下面短文,掌握其大意,然后从A、B、C、D四个选项中选出可以填入空白处的最佳选项。 Dear May, How are you? I’m writing to you at my school. I’m a middle school ______. I am in a big class. There_____ about sixty teachers. My English teacher is Miss Gao. She is a ______teacher .She ______ helps us with English. We go to school ______Monday to Friday .There are four classes in the morning and three in the afternoon .We like to______ between classes. I have some friends at school .One of my friends is from ______. She speaks English. She is friendly ______ us. I usually play with______ .We often play basketball. It is my favorite ______. Please write soon. Lin Tao 36. A. student B. worker C. teacher D. doctor 37. A. am B. is C. be D. are 38. A. bad B. good C. beautiful D. busy 39. A. never B. sometimes C. always D. little 40. A. on B. in C. at D. from 41. A. talk B. talked C. talking D. talks 42. A. English B. UK C. England D. Englishman 43. A. with B. for C. after D. to 44. A. him B. them C. they D. she 45. A. lesson B. color C. game D. subject Ⅶ. 阅读理解(共15小题;第51—60题每小题1分,第61—65题每小题2分,满分20分) 阅读下列短文,然后从A、B、C、D四个选项中选出能回答所提问题或完成所给 句子 的最佳选项。 (A) Do you know Kurt Wenner? Do you like paintings? There are a lot of street artists in the world. People like street paintings very much. 53-year-old Kurt Wenner is one of them. And he is good at 3D street paintings and is the first person to do 3D street paintings. Kurt was born in the USA. He likes art very much. His painting Spider-Man《蜘蛛侠》 is a great success. 46. How old is Kurt Wenner? A. 25 B. 53 C. 52 47. Kurt Wenner is the ___________ person to do 3D street paintings according to the passage. A. first B. third C. second 48. What does Kurt Wenner like very much? A. science B. music C. art 49. Where was Kurt Wenner born? A. the UK B. England C. Italy D. the USA 50. Which is true according to the passage? A. Kurt Wenner is a teacher. B. There are some street artists in the world. C. Kurt Wenner’s painting Spider-Man《蜘蛛侠》 is a great success. D. People don’t like street paintings. (B) All students should have good study habits. When you have good study habits, you can learn things quickly. You also remember them easily(容易地). Do you like to study in the living-room? This is not a good place because it is usually too noisy. You should study in a quiet place, like your bedroom. A quiet place will help you only think about one thing, and you will learn better. Before you begin to study, do not forget to clean your desk. A good desk light is important, too .You’ll feel tired easily if there is not much light . 51. When you have good study habits, you will _____________. A. learn things quickly B. remember things easily C. think about one thing D. both A and B ’s the meaning of the underline word “noisy”? A. 安静的 B.繁忙的 C.热闹的 D.吵闹的 53. Which place is good for study according to the passage? A. the living room B. the kitchen C. the bedroom D. the restaurant 54. You will feel tired easily if there is ______________ light. A. good B. much C. not much D. nice 55. The best title (标题) of this passage is _______________. A. Study in the Bedroom B. Good Study Habits C. How to study D. Desk Light is Important (C) November 9 is a time for us to learn a lot more about fire(火). This is what to do in a fire: out. Shout as loudly as you can, because people may be asleep.? 119. Never try to put out a fire yourself. Tell 119 where you are and what is on fire.? down close to the floor. There is less smoke down there, so it’s easier to breathe and see where you are going.? the door. If the door is cool, open it carefully. If the door is hot, do not open it! Try to find a different way out.? out. Do not stop to pick up anything. A fire can become very big in a few seconds!? ’t use the lift. Always use the stairs. The lift may go wrong and keep you inside.? ’t go back .Even if you have left your pet or favorite toy inside, do not go back for it. Animals have a very good sense of smell. They often get out of buildings before people.? is the National Fire Prevention Day(消防日)? A. December 9 B. November 9 C. November 11 D. September 11 57. How many things should we do according to the passage? A. six B. eight C. seven D. five 58. According to the passage, when something is on fire, you must ________at once. A. look for your pet B .telephone 119 C. call 120 D. put out the fire yourself 59 You should keep down close to the floor because_________. A. there is less smoke down there B. the firemen can find you easily C. people may be asleep D. you should look after your things carefully 60. Which of the following is not right according to the passage? A. If the door is hot, try to find a different way out. B. Don’t use the lift. C. People often get out of buildings before animals. D. Do not stop to pick up anything. Ⅷ. 任务型阅读(共5小题;每小题1 分,满分5 分) Activity Time Place Music Festival Oct. 19th Gym(体育馆) School trip Oct. 25th Yongor zoo Basketball Game Nov. 2nd Playground English Party Dec. 10th Classroom 7E Book sale Gym(体育馆) 请为以下活动选择正确的时间 61. School trip A. Oct. 19th 62. English Party B. Oct. 25th 63. Music Festival C. Nov. 2nd 64. Basketball Game D. Dec. 10th 请仔细阅读,选出最符合要求的最佳选项 many activities are there in the gym? A. five B. one C. three D. two 试题卷II(满分30分) Ⅸ. 词汇运用(共15小题;每小题1分,满分15分) (A)根据下列句子及所给汉语注释,写出空缺处各单词的正确形式,每空只写一词。 66. We have something fun for you this _______ (学期) . 67. Kate and Ann are good friends. They go to the _______ (相同的) school. 68. For _______ (早餐) ,I eat eggs, bananas and apples. 69. Tom _______ (需要)a pen, a notebook and his science book now. 70. I _______ (真正地) like the wonderful basketball match! (B)在下列各句的空格内填上一个适当的完整的单词,该词的首字母已给出。 _______for your help! 72. They ask many q _______ about our school art festival. 73. L_______! This is his ID card. 74. Saturday is the s_______ day of a week. 75. Jeremy Lin is a famous basketball star and he plays basketball w_______ . (C)根据短文内容,用方框中所给单词的适当形式填空,每词限用一次。 Here is a ___76______. A man and a boy are in it. ___77___ are they? Let me tell you. The man is Mr. White, and the boy’s name is Bill. He’s the man’s son. Now __78____are in Bill’s bedroom. We can see some __79_____ books in the desk. And on the bed is a shirt. It ’s Bill’s .His pants are___80____ his shirt. Ⅹ.按要求完成下列句子,每空一词(共5小题;每小题1分,满分5分) 81. This is my sister. (变为一般疑问句) ________ this ________ sister? has eight pencils. (改为否定句) He ________ ________ eight pencils. I help you? (改为同义句) ______ can I do ______ you? 84. I like my computer teacher because she is fun. (根据划线部分提问) ______ ______ you like your computer teacher? 85. eating, star, ask, the, volleyball, habits, I, about, her(连词成句) ________ . Ⅺ. 书面表达(满分10分) 美国男孩Eric来信想在北京征询笔友。假如你是Liu Xing,你想和Eric成为笔友。请根据下面表格中所提供的信息,用英语给Eric写一封回信,介绍你自己的情况。信的开头和结尾已给出,字数不计在内。要求:字迹工整清楚,60字左右。 Name: Liu Xing Age:13 Live in: Beijing Date of birth :November 19th, 1999 A sports collection: 3 soccer balls 5 basketballs 6 volleyballs Favorite food: fruit/healthy Hobbies: play basketball / listen to music Favorite subject : science / difficult but interesting Dear Paul, Thanks for your letter. I know you want a pen pal in Beijing. I want to _______________________________________________________________________ _______________________________________________________________________ _______________________________________________________________________ Please write to me soon. Yours, Liu Xing 听力材料录音稿: Ⅰ. 听小对话, 选择图片。本题共有五个小题,在每一小题内你将听到一个小对话,我们把对话念一遍。请你从试卷上的A、B、C三个选项中,选择一幅恰当的图片。(现在你有15秒钟的时间阅读第1至5五小题) 1. M: What does Bill like? W: He likes volleyball and basketball. But he doesn’t have a volleyball and he only has a basketball. (停顿10秒) 2. M: How many people are there in your family? W: Six. They are my grandparents, my parents, my sister and I. (停顿10秒) 3. W: Hi, Bob. What do you get on your birthday? M: I get lots of things. And the I-pad is the best thing. (停顿10秒) 4. M: What does your uncle do? W: He is a cook. But he wants to be a basketball player. (停顿10秒) 5. W: What animal do you like? M: I like dogs. What about you? W:Cats. (停顿10秒) Ⅱ. 听小对话,回答问题。本题共有五个小题,在每一小题内你将听到一个小对话,我们把对话念一遍。请你从试卷上的A、B、C三个选项中,找出能回答这个问题的最佳选项。(现在你有15秒钟的时间阅读第6至10五小题) 6. M: Where’s the ping-pong bat, Lucy? W: Peter has it. Ask him for it. (停顿10秒) 7. W: Could I speak to May, please? M: Wrong number. This is 7045386. (停顿10秒) 8. W: What a nice T-shirt! How much is it? M: Oh, it’s only 80 yuan. (停顿10秒) 9. W: I like this color-brown. M: I don’t like brown. I like purple and black. (停顿10秒) 10. W: Can I help you? M: I want to buy some history books. (停顿10秒) Ⅲ. 听长对话,回答问题。本题共有两段较长的对话,我们把对话念两遍。请你根据对话内容从A、B、C三个选项中选出一个最佳选项。 听下面一段较长的对话,回答第11至12两小题。 (现在你有8秒钟的时间阅读第11至12两小题) (停顿8秒) W: Hello, I’m Cathy Brown. What’s your name? M: My name is David Green. Nice to meet you! W: Nice to meet you, too! What is your e-mail address? M: It’s about yours? W:Oh, I have an e-mail address, but now I forget it. (停顿5秒后, 朗读第二遍) (停顿10秒) 听下面一段较长的对话,回答第13至15三小题。 (现在你有12秒钟的时间阅读第13至15三小题)) (停顿12秒) M: Mum, What a big box on the table? What’s in it? W: Guess! M: A model plane? W: Yes. I think you’ll like. M: Wonderful! Is it for my sixth birthday? W: Sure, happy birthday to you! M: Thank you, mum. (停顿5秒后, 朗读第二遍) (停顿15秒) Ⅳ. 听短文,回答问题。本题是一篇短文,短文后有五个问题,我们把短文念两遍。请你根据短文内容从A、B、C三个选项中选出一个最佳选项。 (现在你有15秒钟的时间阅读第16至20五小题) (停顿15秒) Tina and Lisa are middle school students. They want to go shopping. And they don’t have classes on Sunday. So they go to the store. It is a very nice store. The things are all at good prices. Tina buys a pen and Lisa buys a pencil case. In the store, there is also food. Tina buys some apples and bananas. Lisa buys some tomatoes and carrots for her mother. They are very happy to go back home with so many things. (停顿5秒后, 朗读第二遍) 七年级英语期末考试题参考答案 66. term 69. needs 72. questions 73. Look 74. seventh 76. picture 78. they 79. Chinese 80. under your ’t have for do 85. I ask the volleyball star about her eating habits. Ⅹ. 书面表达(共15分) One possible version: Dear Eric, Thanks for your letter. I know you want a pen pal in Beijing. I want to make friends with you. My name is Liu Xing. I’m a 13-year-old boy. I live in Beijing. My birthday is on November 19th, love sports. I have a big sports collection. I have 3 soccer balls 5 basketballs and 6 volleyballs. And I often play sports with my friends after school. 七年级英语期末考试题相关 文章 : 1. 七年级上册英语期末试卷及答案 2. 2015-2016初一英语期中考试题 3. 七年级第一学期英语期末测试题 4. 七年级英语上册期末模拟试题 5. 七年级上册英语期末复习

英语是学生主要的学科之一,分享了初一英语试题答案给同学们学习!

I.听力理解 (20分)

(Ⅰ)录音中有五个句子, 听一遍后, 选择与之相符的.图片。(5分)

(Ⅱ)录音中有五组对话及五个问题, 听两遍后, 选择最佳答案。(5分)

( )6. A. It’s in front of the school. B. It’s behind the park. C. It’s in front of the park.

( )7. A. At 9: 00. B. At 9: 15. C. At 9: 45.

( )8. A. Yes, he is. B. No, he isn’t. C. He is a worker.

( )9. A. Tea. B. Juice. C. Water.

( )10. A. Tom’s uncle. B. Tom’s father. C. Tom doesn’t know him.

(Ⅲ)录音中有两段对话, 听两遍后, 选择最佳答案。(5分)

听第一段对话, 回答第11、12小题。

( )11. Where are Tommy and Tony from?

A. They are from England. B. They are from America.

C. They are from China.

( )12. What do they like?

A. They like playing basketball. B. They like playing football.

C. They like playing basketball and football.

听第二段对话, 回答第13~15小题。

( )13. What does the woman want to buy?

A. A doll. B. A T-shirt. C. A cake.

( )14. What colour does her daughter like?

A. Blue. B. Green. C. Red.

( )15. How much is the T-shirt?

A. 25 yuan. B. 50 yuan. C. 15 yuan.

(Ⅳ)录音中有一篇短文, 听两遍后, 选择最佳答案。(5分)

( )16. I am .

A. in schoolB. Twelve C. thirteen

( )17. Sally is .

A. a teacher B. a student C. a doctor

( )18. Sally’s schoolbag is .

A. newB. oldC. on the bed

( )19. The desk in the room is .

A. blackB. white C. blue

( )20. is a bus driver.

A. SallyB. Sally’s father C. Sally’s mother

Ⅱ. 单项选择 (每小题1分,共15分)

( )21. Jane is . She is from .

A. England; English B. an English; England

C. English; EnglishD. a English; England

( )22. There isn’t beef in the fridge. I must buy today.

A. some; someB. any; any

C. any; someD. some; any

( )23. —What’s your name, please?

A. I’m a teacher. B. His name is Paul.

C. How do you do?D. I’m Li Lei.

( )24. I don’t like running, I like swimming.

A. and B. but C. with D. so

( )25. — this your family?

—Yes, these are my parents and those my brothers.

A. Is; areB. Are; areC. Is; isD. Are; is

( )26. interesting story it is!

A. What anB. WhatC. How anD. How

( )27. —What’s this in English?

— a computer.

A. This isB. It isC. That isD. They are

( )28. Betty is in Jinan her parents.

A. and B. with C. for D. to

( )29. —Have you got any fruit, Helen?

A. Yes, I am. B. No, I don’t. C. Yes, I have. D. No. I hasn’t.

( )30. — do you have for dinner?

—A big hamburger.

A. WhoB. WhatC. WhereD. When

( )31. Look at the big room. It’s .

A. Eve and AnnB. Eve’s and Ann

C. Eve and Ann’sD. Eve’s and Ann’s

( )32. —Are there any shops near your home?

—Yes, .

A. it isB. they are C. there isD. there are

( )33. —Is your father a teacher or a worker?

A. Yes, he is. B. No, he isn’t. C. A worker. D. I don’t know.

( )34. They TV in the evening every day.

A. are watchingB. watch C. watchedD. watches

( )35. — ?

—It’s Monday.

A. What’s thisB. What day is it today

C. How’s the weatherD. What’s the time

Ⅲ.完形填空 (每小题1分,共10分)

This is a photo of Mr. King’s family. Mr. and Mrs. King 36 two sons and a daughter. They are in England. Mrs. King is 37 Chinese teacher. Mr. King is a bus driver. 38 daughter is eleven and 39 is a student. Her name is Kathy. She is in Class Four 40 No. 1 Middle School. There are thirty students in her class. Tom and David 41 her brothers. They are nine years old. They are 42 . They are 43 , too. They go to school at 7: 30. They have 44 in the morning and two in the afternoon. Tom and David like 45 and they play it every day. They love their sister very much.

( )36. A. there areB. there isC. has gotD. have got

( )37. A. aB. anC. theD. /

( )38. A. TheyB. ThemC. TheirD. Our

( )39. A. heB. hisC. sheD. her

( )40. A. inB. forC. onD. with

( )41. A. beB. amC. isD. are

( )42. A. youngerB. olderC. biggerD. smaller

( )43. A. friendsB. brothersC. studentsD. boysw w

( )44. A. breakfastB. four classes

C. some breadD. a break

( )45. A. schoolB. teachers C. ChineseD. football

Ⅳ.阅读理解(每小题2分,共20分)

A

There are three people in my family. They are my father,my mother and me. My father is a teacher. He teaches English. My mother is a teacher, too. She teaches Chinese. I am a student. I’m in Class Two. I like my father and my mother and my parents like me very much(非常).

( )46. How many people are there in my family?

. . Four.

( ) is my father?

. . Teacher.

( ) class am I in?

. Class . Class Four.

( ) does my mother teach?

. . Maths.

( ) my mother like me?

, she . No, she doesn’. Yes, she doesn’t.

B

My name’s Tony and I’m English. These are my parents. My mother is an English teacher in a university in Beijing. This is my father. He’s a hotel manager.

My name’s Li Daming. I’m Chinese. These are my parents. My mother is a doctor at the hospital. My father is a factory manager.

( )51. Tony is _________.

A. English B. England C. China

( )52. Tony’s mother is _________.

A. an English teacher B. a hotel manager C. a doctor

( )53. –Is Tony’s father a hotel manager? - __________

A. Yes, he is. B. No, he isn’t. C. He is a teacher.

( )54. In English “工厂经理” is __________

A. hotel manager B. factory manager C. manager

( )55. What’s Daming’s mother?

A. a work B. a teacher C. a doctor

Ⅴ. 词汇运用 (10分)

(Ⅰ)根据句意及首字母或汉语提示完成单词。(5分)

56. I like eating m , like chicken and beef.

57. There are seven days in a w .

58. —Where’s your mother?

—She is making cakes in the k .

59. Eating a lot of (蔬菜)is good for our health.

60. —What’s your father?

—He is a (农民).

(Ⅱ)用所给词的适当形式填空。(5分)

61. Tomorrow (be)my birthday. I’ll have a party.

62. Let’s (buy)her a new phone.

63. There are a lot of (potato)in the bag.

64. (this)are my English story books.

65. Lucy is the (one)student to get full marks(得满分).

Ⅵ.句型转换 (每小空1分,满分10分)

66. Are there any books in the bag? (作肯定回答)

__________,there________.

67. His pen is on the desk.(对画线部分提问)

________ _________ his pen?

68. Lingling is twelve years old. (对画线部分提问)

_______ _______ is Lingling?

69. There are some flowers on the table.(改为否定句)

There_________ _________ flowers on the table.

70. 大明在哪儿?(翻译成英语)

__________ __________ Daming?

笔试部分

II.单项选择。(每小题1分,共20分)

从A、B、C、D四个选项中选出可以填入空白处的最佳答案,并把答题卡上对应题目的答案标号涂黑。

( ) in the classroom?

( ), I like milk and eggs.

( )31.—Can I help you?

—________

, .

( )’ day is on ________.

( ) is ________Chinese boy’s name is Xiaoqiang.

; A ; An ; The ; The

( ) science class is ________ Monday________Wednesday.

; to ; to ; for ; about

( )35.—________

—She is my English teacher.

’s she? she your teacher?

’s she? is she?

( )36.—What do you want?

—I want some ________ and________.

; chickens ; chicken

; chicken ; chickens

( ) is a________ teacher, and he also plays soccer ________.

; good ; well ; good ; well

( ) are not ________, we do not eat ________.

; they ; they ; them ; them

( ) my classes finish _____ 5:00 ..

( ) and I ________ in ________ class, we are good friends.

; same ; same ; the same ; the same

( ) ________ all fruit but I ________.

; don’t ; don’t ; not ; aren’t

( ) grandpa always________ us ________sports.

; play ; play ; plays ; plays

( )43.—How much is the quilt?

—________

’s 15 ’re 15 ’s 15 's an orange.

( ) our great sale, we have skirts ________ red ________ 20 dollars.

; for ; in ; for ; in

( )45.—______ you _____ your help, Linda.

—You’re welcome.

; for ; of ; for ; about

( )46.—When _________he have

—He ________ it on Monday.

; has ; have ; have ; has

( )47.—Happy birthday!

—________

, birthday! .

( ) English test is difficult ________ interesting.

III.完形填空。(每小题分,共15分)

根据短文内容,从A、B、C、D四个选项中选出一个能填入相应空格内的最佳答案,并把答题卡上对应题目的答案标号涂黑。

Dear Claire,

You ask me about my favorite is Spring (春) is 49 January or the festival, we do many things and buy many things, so we are always 50 .The 51 of Spring Festival is 52 .We wear (穿)new red 53 and put up (张贴)red paper-cuts (窗花).And we put (放)lucky money (压岁钱)in red packets (红包).Boys and girls are always happy those days, 54 they can have a good time with 55 family, and they can get lucky money from their parents, grandparents, uncles and do we 56 on Spring Festival? Jiaozi and some the evening, the family will watch TV together, but children(孩子们)57 watching TV for 4 ’s really boring for always play 58 their friends.

What about you? What festival do you like best?

Your friend,

Li Ju

( )

( )

( )

( )

( )

( )

( )

( )

( ) ’t like

( )

IV.阅读理解。(每小题2分,共30分)

阅读下列材料,从A、B、C、D四个选项中选出最佳答案,并把答题卡上对应题目的答案标号涂黑。

A

Spring Festival(春节)comes on February 19th this year! Boys and girls like it very much because(因为)they don’t go to school from February 1st to February they can have new clothes and much can also eat lots of in Liangshan, Sichuan, some boys and girls are not don’t have warm(暖和的)clothes or can’t go to need you want to help them? Call us at 87865866.

Festival is on ________.

1st 19th 26th 9th

(大多数的`)boys and girls can have________during Spring Festival(春节期间).

clothes soccer ball telephone

Sichuan, some boys and girls don’t have ________.

clothes and C

B

Students like fast food(快餐)very of them like to meet friends in a fast food can eat many kinds(种类)of food and listen to(听)music in the like to have chicken, French fries, hamburgers, salad and favorite food is I don’t think fast food is like vegetables and are healthy food.

like ________very much.

cream food

does “restaurant” mean in Chinese?

A.商店 B.餐馆 C.书店 D.酒店

are healthy food.

fries and chicken and Cola

and fruits and ice cream

favorite food is _______.

C

Hi, boys and girls! A new(新的)Mama store is in Middle School and see at our great sale! We have bread for 3 milk tea is on sale for 2 hamburgers are only 3 you need notebooks ? They are only 6 yuan for also have pens in all colors, only 1 yuan each(每支).Some students want to buy have black socks for only 10 yuan for 3 girls, we have socks in pink and purple, they are very and buy your things at our great sale.

new store is ______.

the school our room n the library the classroom

wants to buy a notebook and a pen, she pays(付钱)________.

yuan.

can’t buy ________ at the store.

article (文章)is ________.

letter(信件) notice(通知) e-mail ad(广告)

D

This is the timetable for the tests of Chongqin g Middle ’s for the students

in Grade(年级) the tests, their first term in this school will finish.

Dates Days Time Subjects

January 26th Monda 9:00 - 11:00

2:30 - 4:30

January 27th Tuesday 9:00 - 11:00

2:00 - 3:30 and history

students have ________ tests in the morning.

and history and math

Chinese for t he word “timetable” is ________.

A.座次表 B.课桌 C.时间表 D.饭桌

students have tests of ________ subjects.

of the following(以下哪句)is RIGHT?

students have an English test for 2 hours.

students have a geography test.

math test is on .

Ch inese test is in the afternoon.

第Ⅱ卷(共 55分)

V.任务型阅读。(每小题2分,共6分)

阅读下文并回答问题。

Dear friends,

Today(今天)is a great ’s my you come to my birthday party?

I have some nice and fun things for have a big ball collection(收藏), because I like sports very we can eat a big birthday cake(蛋糕)then.

Let’s meet at 6:00 in the afternoon, in our classroom.

Please call me at 68314567.

Cindy

Cindy’s birthday on January 12th?

______________________________________________________________________________

can we eat in the party?

______________________________________________________________________________

does Cindy like balls?

______________________________________________________________________________

VI.口语交际。 (每小题1分,共5分)

阅读下面对话,从方框内7个选项中选择5个恰当的句子完成此对话。请填写字母。

Grace: Do you like watching TV?

Jane: 77

Grace: 78

Jane: I AM A SINGER.(歌手)

Grace: Do you like Han Hong?

Jane: No, I don’ I like Zhang sings very

Grace: I like THE EMPRESS OF CHINA(武媚娘传奇).The girls in it are very beautiful(漂亮).

Jane: 80

Grace: It’s from 7:30 to 10:00 ..

Jane: 81

, I don’t.

, I do.

about you?

’s your favorite show(节目)?

, let’s watch TV this evening.

When is it?

’s for sure!

___________

VII.用所给单词的正确形式填空。(每空1分,共6分)

don’t like hamburgers, I don’t want ________ (be)fat.

are good for you, ’s ________ (have)some.

is the ________ (nine)month of a year.

’s ________ (use)to buy a dictionary.

is easy for ________ (I).

always watch RUNNIG MAN on TV, and I ________ (real)like it.

VIII.完成句子。(每空1分,共12分)

根据所给提示,完成句子。每空一词,含缩略词。

have a basketball.(变为一般疑问句)

__ ______ they ________ a basketball?

daughter is only 8 months old.(对划线部分提问)

________ ________ is your daughter?

90.体育明星都有良好的饮食习惯。(完成译句)

Sports stars have good ________ ________.

favorite subject is ..(对划线部分提问)

________ ________ favorite subject?

am not free on Monday.(同义句转换)

I am ________ on Monday.

geography class is at 8:00.(对划 线部分提问)

________is the geography class?

father thinks the teachers are right.(变为否定句)

His father ________ ________ the teachers are right.

IX.短文填空。(每空2分,共16分)

根据下面短文内容,在短文的空格处填上一个恰当的词,使短文完整、通顺。每空一词, 注意在文中的大小写形式。

Hello! My name is am 95 American Favorite day is 96 because the next day is I don’t need to go to Friday, after I finish my homework(家庭作业), I can go to the park with my ’s a 98 time(时光)for like to play tennis with my father, because he plays it 99 .100 my mother doesn’t play 101 , she only watches the evening, we go to 102 some fruit in the like bananas and oranges very much.

X.书面表达。(共10分)

学校“男神女神墙”上榜人物评选活动正在火热进行中,请你推荐一个心目中的人选(可以是同学、老师或自己)。

内容包括:

1、姓名、年龄。

2、体育爱好、饮食习惯(各举至少一个例子)。

3、喜爱的(或任教的)学科。

注:书写工整、卷面整洁,文中不得出现真实姓名及班级。(60词左右)

Hello, everyone, I have a right person(合适人选)for this “wall(墙)” .___________________

II.单项选择:

29-33 A A B D D

34-38 A C C D C

39-43 B C A B A

44-48 A C D B B

III.完型填空:

49-53 D A B D A 54-58 D B A C A

IV.阅读理解:

第II 卷 (共55分)

V.任务型阅读:

, it is.

bir thday cake.

she likes sports (very much).

VI.口语交际:

77-81 B D C F E

VII.单词正确形式填空:

be

VIII.完成句子:

IX.短文填空:

good/ great/ nice

定语从句:

(一)基础:引导词---who, which, that, whom,

练习 like cities___________are quiet and prefer students _________are hard- working.

hate TV shows _________ are noisy and music __________ is gentle and quiet attracts me a lot.

food __________tastes delicious is not always healthy.

boys ___________ are playing basketballs over there are from Class Fifteen.

books ____________ are written by Lu Xun are worth reading.

town ___we visited last week is much larger than book ___ he bought is very interesting.

(二)特例:只用that的情况

1先行词被___________或___________所修饰,或本身是______________________时,只能用 that,

2.被修饰的先行词为 ________________________________________________等不定代词时, 只能用 that.

3.先行词被 _____________________________________________等词修饰时,只能用 that,而不用 which。

4.先行词里同时含有______________________,如I can remember well the persons and some pictures that I saw in the room.

5.以______________________引导的特殊疑问句,只能用that.如: Who is the girl that is crying?

练习 am interested in everything___ is about the 2006 World Cup.

there anything___I can do for you? is the very bike ______I lost

my surprise, he gave me nothing __ I is the best dictionary __ I have ever used.

was the first person _______passed the exam.

talked happily about the men and books________interested him greatly.

(三)whose

student ______father works in the factory is sitting there.

like the rooms ______windows face is the desk ______legs were broken.

4The woman ___ umbrella you took is angry about comes a girl ___ handwriting is the best .

6The banana __skin is green can’t be tall tree _ leaves are yellow is very old.

(四)从句谓语单复数由先行词确定,时态由从句时间状语决定,不必跟主句保持一致。

1-He is one of the boys who ____ (doesn’t, don’t)finish doing homework.

2-I like films which ____(be)exciting and interesting.

3-Children who often ____(eat)junk food are easy to become fat and unhealthy.

4-That boy who _____(run)fastest is from our class.

5Those boys who ____ playing the guitar are from our school.(be)

6The trees which _____ (be)watered yesterday belong to man who ____ over there is our teacher.(stand)8Those boys who ____ playing the guitar are from our school.(be)

is one of the foreign experts _______ _________ (work)in China.

10I’m one of the boys ______ ________(like)English best.

(五)“介词+关系代词”注意: 介词的选用要考虑:

A.与先行词的搭配关系

1)I will never forget the day __________ I joined the army.

2)I will never forget the days ______ I worked here.

3)I will never forget the year ____________ my son went to college.

B.与谓语动词的搭配习惯

1)Have you found the book ________I paid 29 dollars?

2)Have you found the book ________I spent 29 dollars?

3)Have you found the book _____________we learnt a lot?

4)Have you found the book _______she often talks?

关系副词的用法

练习: must be a good place ________________we can do a lot of exercise.

is the village _________________we visited last week.

house _______ we live in is very big./ The house _______ we live is very big

woman ___________ talked to you just now is a woman ______ you talked to is my sister.

The woman to ___________ you talked is my is the hospital ____________ I was born in.

This is the hospital in _________I was is the hospital____________ I was born.

综合练习一.用适当的关系代词或关系副词填空

boy ______ is wearing the black jacket is very clever.

is the present ____he gave me for my man _______ talked to you just now is an engineer.

talked about the teachers and schools _______ he had visited.

is nothing in the world ______can frighten visited a factory _______makes toys for children.

this the place _______ your father once lived ’ll never forget the days _______ I joined the League.

9The car______my father bought last month is man______hair is white is his grandfather.

初一英语考试试卷

七年级期中考试就要到了,为让同学们对英语期中考试有更好的准备,那就要多做几份英语期末试卷吧。以下是我给你推荐的七年级英语期末试卷及答案,希望对你有帮助! 七年级英语期末质量检测试卷 一、听力部分 (一)录音中有五个句子,每个句子听一遍,然后从每小题A、B、C中选出能对每个句子作出适当反应的答语。(每小题1分,共5分) , they are. , they aren't. 'd like to. , it is. , it's big. , it isn't. 7:30 bicycle my friend , he was. was born in Shandong. was born in 2000. , you were. , I was. , I didn't. (二)录音中有五组对话,听对话两遍后,从每小题A、B、C中选出能回答所给问题的正确答案。(每小题1分,共5分) could swim. could ride a horse. could ride a bike. will send their homework by letters. will send their homework by themselves. will send their homework by email. , he did. , he didn't. , he doesn't. . . . (三)录音中有一组对话,听对话两遍后,从每小题A、B、C中选出能回答所给问题的正确答案。(每小题1分,共5分) story happened ________. year ago. in the future. long ago. did Nuwa die? ________ Yan pushed her into the river. jumped into the river. boat overturned(翻) when she rowed(划) it. changed into a ________ father was very __________ after Nuwa died. the end, the river was ________ there (消失) than before (注意:请同学们翻到第Ⅱ卷第四大题,继续作听力填空题。) 二、选择填空(每小题1分,共15分) ( ) What are you doing now? -- I am __________ my clothes, Mum. dressed on ( ) maths problem is __________and I can do it _________. , easily , easily , easy , easy ( )'m looking forward to _______ to Disneyland __________ my parents. , with , with , and , with ( ) usually goes to school____ foot or________ a bike. , by , on , on , by ( )'s the cheapest way to get________ Shanghai? , to , to , to , in ( ) ______ is it from Weifang to Dezhou? --- More than 300 kilometres. many much long far ( ) was a pond _______a lot of beautiful fish _______ it. , in , in , with , with ( ) your father and mother born in 1965? --- Yes, they _______. But my father ________born in Beijing. He ____ born in Tianjin. , were, wasn't, was , are, isn't, is , were, weren't, was , were, was, wasn't ( )’t point _________the old. ( ) _________ the school when the bell rang. to to in ( ) his work was over, he________ his home. to to back ( ), there is a book on the floor. Can you _________ ? it up up it them up up them ( ) girl ________ Lacy is my good friend. called called ( ) I was a student ten years ago, I _______ many English books every day. at at ( ) is _________ girl on the bus. 8 years old 8-year-old 8 years old 8-year-old 三、阅读理解。(每小题2分,共30分) A Today more and more students wish to get more think money is they have a lot of money, their dreams will come can do everything with can get many story books and MP4s and play games on can get what they course,money is very important to will feel happy when they listen to MP4s and play order to get more money,maybe some of them will steal or rob(偷或抢)or do something they must know money can’t bring them can’t buy happiness,knowledge or health with money. and more students wish to get more _________. books students have more money,___________. can buy everything can buy happiness can buy time can buy story books can’t buy________. of the above order to get more money, maybe some students _________. make money get money from parents do something wrong save money is the main idea(中心思想)of the passage? think money isn’t everything. can’t buy everything. money students can do nothing. and more students wish to he healthier B Name Personal information Yuan Longping born On September 7th,1930,Beijing,China,scientist Bill Gates born on October 28th,1955,Seattle,the USA,the founder of Microsoft YaoMing born on September l2th,1980,Shanghai China,super sports star Stephen Hawking born on January 8th,l942,Oxford,Braitain,professor Jay Chou born on January 18th,l979,Taiwan,China,super music star was born in October? Chou Hawking Gates Longping was Stephen Hawking born? Seattle Oxford Beijing Shanghai old is Yao Ming now? is the oldest of the five people? Hawking Gates Chou Longping of the following is TRUE? Gates was born in Oxford, Braitain. Longping is a super sports star. Chou is a super music star from Taiwan, China. Hawking is the founder of Microsoft. C The Greens live in a very nice city. They live next to the Whites, but the Greens want their life to be better than the Whites' . They have a bigger television, a bigger fridge and even brighter shoes. "Little White has a new bicycle. It goes faster than yours. Tomorrow I will buy you a better bicycle." "Mr. White has a big fish. I' m going to the market to buy a bigger one." "Mrs. White has a new chair. I' m going to get a bigger, stronger and prettier one. My life is going to be better than hers." A friend asks Mrs. White, "Do you think the Greens have a better house than yours?" Mrs. White says, "The Greens? Oh, the people next door. I don't think about them much. They always look a little sad. I think we have much better life and home. We are happier. We have more fun and our family life is warmer." 根据短文内容,判断下列句子正(T)误(F)。 Greens want their life better than the Whites. Whites' TV is much better than the Greens' . White has a new bicycle. . White isn't interested in the Greens. the story we can know the Whites have much happier life than the Greens. 卷Ⅱ(共60分) 四、听力部分 录音中有一篇文章,听文章两遍后,根据其内容,填写下面的短文。(每空最多3词,共5分) Once 46_________a man who lived with 47__________Papa day, he fed bananas to these one day, he almost ran 48__________the he wanted to give them three bananas in the morning and four bananas at the monkeys didn’t Papa Monkey 49___________:four bananas in the morning and three bananas at monkeys thought that was china, if someone changes his mind too often,people will say, “Three in the Morning,Four at Night.” 五、根据句意及首字母提示补写单词,完成句子。(每空l分,共10分) ’s the p_________ of your city? yellow river is the second l____________ river in China. English teacher is f_____________ to all of us. ’t speak loudly, please! The girl is a___________. left w___________ saying a word angrily. . Jackson works in a computer c______________. We all know, Zhan Tianyou is a national h____________. is too fat and he can’t go t____________ the door. Hong’s parents are s___________ with her. are you going to do on h____________ this summer? 六、根据括号中动词的适当形式完成句子。必要时可加助动词或情态动词。(每空l分,共10分) aunt and my uncle _________(meet)me at the airport two years ago. children enjoy___________(listen)to the story. mother often __________(go)shopping on last Sunday she_________ (not go) shopping. fastest way ____________ (travel)is taking a plane. Lei with his father __________(not be)at home last night. Shakespeare _________(be)born in l564 and _________(die)in l616. (buy)a house one year ago. term my son ________(walk)to school because of our moving. 七、阅读短文,回答问题。(共10分) At l4:28 on May l2 2008,an earthquake broke out in Wenchuan,Sichuan lots of people all over China went to help the people PLA saved a boy staying in the debris(废墟)for ten hours after the was three years name is Lang he was lying on the stretcher(担架),he slowly raised his right hand and saluted(敬礼)to the salutation(致敬)not only showed his thanks to the also moved the people around ’m sure it also deeply moved many other a three-year-old boy, he knew that the first thing to do was to show his thanks are people saved is the good nature of Chinese people. did the earthquake in Wenchuan break out? __________________________________________________________ saved a boy named Lang Zheng? ________________________________________________________ did Lang Zheng do first after the PLA saved him? ________________________________________________________ did Lang Zheng’s salutation show? ________________________________________________________ 73 .Did his salutation deeply move you? ________________________________________________________ 八、书面表达。(共25分) A).根据汉语意思完成句子。(每题2分,共10分) 74.昨天骑自行车到那里花了我半小时。 It _________ me half an hour_________ there by bike yesterday. 75.在进入老师办公室之前,你应该先敲门。 You should ___________the door before entering the teachers’ office. 76.当你找不到路时,我们说你迷路了。 When you can’t find your way, we say you ___________. 77.中国以长城而出名。 China is ____________ the Great Wall. 78.在1998年,他与一位漂亮的姑娘结了婚。 He _____________ a beautiful girl in l998. B).根据下面的汉语提示,写一篇短文。(共15分) 鲁迅,现代著名的文学家、思想家和革命家。请根据下面表格中有关鲁迅的生平简介,写一篇不少于50词左右的短文,介绍鲁迅先生. Time Activities 1881 born in Shaoxing,Zhejiang Province 1886 went to school 1902 went to Japan to study 1909 came back and worked as a teacher 1918 began to write novels 1936 died in Shanghai 七年级英语期末质量检测试卷答案 一、听力部分 (一)录音中有五个句子,每个句子听一遍,然后从每小题A、B、C中选出能对每个句子作出适当反应的答语。(每小题1分,共5分) 1—5 AABCB (二)录音中有五组对话,听对话两遍后,从每小题A、B、C中选出能回答所给问题的正确答案。(每小题1分,共5分) 6—10 ACBBB (三)录音中有一组对话,听对话两遍后,从每小题A、B、C中选出能回答所给问题的正确答案。(每小题1分,共5分) 11—15 CCABB 二、选择填空(每小题1分,共15分) 16—20 CABCA 21—25 DAA DC 26—30 BABAD 三、阅读理解。(每小题2分,共30分) 31—35 BDDCB 36—40CBCDC 41—45TFTTT (听力部分) 四、录音中有一篇短文,听短文两遍后,根据短文内容,填写下面的文章。(每空2-3词,共5分) was called of an idea better 五、根据句意及首字母提示补写单词,完成句子。(每空1分,共10分) 六、根据括号中动词的适当形式完成句子。必要时可加助动词或情态动词。(每空1分,共10分) didn’t go travel 't/was not died walk/is going to walk 七、阅读短文,回答问题。(共10分) 14:28 on May 12 2008. PLA. saluted(敬礼)to the soldiers. showed his thanks to the PLA. , it did. 八、书面表达。(共25分) A).根据汉语意思完成句子。(每题2分,共10分) , to get at/on lost for married to B).根据下面的汉语提示,写一篇短文。(共15分)略猜你喜欢: 1. 七年级英语测试题及答案 2. 七年级期末英语检测试卷及答案 3. 七年级英语期末考试卷附答案 4. 初一期末考英语试卷及参考答案 5. 七年级上册英语期末考试卷及答案

初一英语期末考试即将来临,为了帮助同学们备考英语期末考试,尝试着完成这份英语期末试卷吧。以下是我给你推荐的初一英语下册期末试卷及参考答案,希望对你有帮助! 初一英语下册期末试卷 一、单项选择(每空1分,共15 分) ( ) , we can sports in school an hour a day now. ---- Really? That sounds great. A. to play B. plays C. play D. playing ( ) goes to school after breakfast every morning. A. the, the B. \, \ C. the, \ D. \, the ( ) 3. We should do more in our lessons. A. exercise B. exercises C. exercising D. exercised ( ) 4. The fish . A. taste well B. tastes well C. taste good D. tastes good ( ) 5. --- How many teachers are there in your school? --- About four . A. hundred B. hundreds C. hundreds of D. hundred of ( ) 6. She often a train to school, sometimes she goes to school bus. A. takes, takes B. takes, by C. by, takes D. by, by ( ) 7. It will you a long time a language well. A. spend, learn B. to spend, to learn C. take, to learn D. take, learn ( ) 8. Li Tao is from a small village. A. a 11 years old boy B. a 11-year-old boy C. an 11-year-old boy D. an 11 years old ( ) teacher is very strict us our homework. A. in, in B. with, with C. in, with D. with, in ( ) 10. The movie is , and we are really very when we see it. A. relaxing, relaxing B. relaxed, relaxed C. relaxing, relaxed D. relaxed, relaxing ( )11. There are many animals in the zoo. But some animal are scary. A. kinds of, kind of B. kinds of, kinds of C. kind of, kind of D. kind of, kinds of ( )12. There are a lot of in our school. A. woman teachers B. women teachers C. woman teacher D. women teacher ( ) 13. –Is the woman your teacher? -- Yes, she teaches Chinese. A. us B. our C. ours D. we ( ) 14. – Excuse me. Is there a post office near here? -- . A. Yes, it is. B. No, this isn’t. C. Yes, there is. D. No, there aren’t. ( ) 15. – Have a nice weekend. -- . A. That’s OK. B. No problem. C. You’re welcome. D. Thank you. 二、完形填空(每空1分,共10 分) People in different 16 like to eat different foods. 17 kind of food people in the UK 18 to eat? Fish and fried 19 chips are very 20 in the UK. Sometimes people 21 them at home, but they usually go to a fast food 22 to buy them. They can have them 23 the restaurant or 24 them away home. Sometimes, they 25 like to eat them in a park. ( )16. A. country B. countries C. city D. town ( ) 17. A. Who B. When C. What D. How ( ) 18. A. would like B. like C. likes D. enjoys ( ) 19 A. potato B. potatoes C. potatos D. tomatoes ( ) 20. A. cheap B. large C. popular D. dear ( ) 21. A. cooking B. cook C. cooks D. to cook ( ) 22. A. restaurant B. library C. supermarket D. mall ( ) 23. A. in B. on C. under D. from ( ) 24. A. get B. bring C. take D. to take ( ) 25. A. and B. also C. too D. either 三、阅读理解(每空2 分,共30分) A We had a terrible school trip last week. Some students were late. Then we waited half an hour for the school bus, but it didn’t arrive. Finally, our teacher took us to take the subway. It took over an hour. When we arrived at the zoo, we were all tired and hungry. We wanted to see dolphins, but there weren’t any. There were some really smart seals(海豹), but we didn’t see the show because we arrived too late. We forgot our cameras, so we didn’t take any photos. Then it started to rain, and no one had an umbrella. We didn’t see many animals because of the rain. So we went for lunch. So we ate hamburgers. My friends also had some ice—cream. But I didn’t have any, because I don’t like it. I wanted French fries, but there weren’t any in the shop. They were sold out! Finally we went back to school. We were wet, tired and stressed out. I didn’t enjoy my school trip at all. ( ) did the teacher take the students to take the subway? A. Because the students didn’t want to have the trip. B. Because the school bus didn’t arrive. C. Because the students didn’t like taking the school bus. D. Because the students didn’t take cameras. ( ) 27. The students were when they arrived at the zoo. A. excited B. happy C. tired D. lucky ( )28. What animal show was there at the zoo? A. Dolphin show. B. Seal show. C. Monkey show. D. Panda show. ( )29. How was the weather after they arrived at the zoo? A. Sunny. B. Cloudy C. Windy D. Rainy ( )30. What did the writer think of the school trip? A. Terrible. B. Great. C. Not bad. D. Fun. B It’s easy to get lost(迷路) in strange city. Here are some tips(方法) to help you find the way and get to your destination(目的地). Take a map with you when you go to a new place. If you lose your way, look at the map, you’ll find the place where you want to go. If you still can’t find the place on the map, ask the police for help. They will tell you the right way to go. Remember the name of your hotel and the street where it is. If you can’t find the way back, take a taxi. I hope these tips can help you. ( ) 31. What does the underlined word “strange” mean? A. 熟悉的 B. 美丽的 C. 现代的 D. 陌生的 ( ) 32. How many tips are there in the passage? A. One. B. Two. C. Three. D. Four. ( ) 33. What should you take with you when you go to a strange city? A. A taxi. B. A map. C. A watch. D. A dictionary. ( ) 34. You should ask for help if you lose your way. A. a man B. a woman C. the police D. your teacher ( ) 35. Which is the best title(标题) of the passage? A. Get lost in a large city B. Some useful tips C. Never go to a strange city D. A map is useful C Hi, I’m a middle school student. I’m Judy. In my class there are thirty-eight students. How do they go to school every day? Seventeen of them take buses because they live far from the school. Six students walk. They think walking is good exercise. Fifteen students ride their bikes to school because they like riding bikes very much and their homes aren’t far from the school. No one goes to school by car. How do I go to school? Ha, I’m one of the fifteen. ( ) 36. There are students in Judy’s class. A. 29 B. 30 C. 38 D. 45 ( ) 37. Six students think is good exercise. A. running B. walking C. swimming D. riding ( ) 38. students ride their bikes to school. A. 17 B. 6 C. 10 D. 15 ( ) 39. Judy goes to school . A. on foot B. by bike C. by bus D. by car ( ) 40. No one gets to school . A. by car B. by bike C. on foot D. by bus 四、选用方框里的句子补全对话(每空1分,共5分) A: Hi, excuse me. ( ) B: Yes. 41 . ( ) A: Well, I’m new in town. 42 ( ) B: Yes, there is. It’s on Bridge Street. 43 ( ) A: Oh••• 44 B: It’s not too far from here. I can walk with you. A: Oh, that’s great! Thanks so much. ( ) B: 45 五、句型转换(每空2分,共10分) 46. You can’t wear sunglasses in the classroom.(改为祈使句) sunglasses in the classroom! 47. My weekend was boring.(对划线部分提问) your weekend? 48. Li Ming did his homework last night. (改为否定句) Li Ming his homework last night. 49. How’s the weather? (同义句转换) the weather ? 50. Joe sits on my left. Lucy sits on my right. (合并为一句) I sit Joe Lucy. 六、词语运用。(每空1分,共10分) (A)根据句意及首字母提示完成单词。 51. N and ten is one hundred. 52. Everyone must follow the school r . 53. If you want to borrow(借) a book, you can go to the l . 54. The house is too e . We can’t buy it. 55. Liu Huan is a famous m . I love his songs. (B)用所给词的适当形式填空。 56. I enjoy (skate)on the ice in winter. 57. Many children like to eat (candy). 58. At the zoo, Jane (take) many photos along the way. 59. Look! Mike (use) a computer . 60. It took me an hour (do) my homework. 七、 完成句子。(每空1分,共10分) 61. 我们必须准时到达学校。 We must get to school . 62. 我的父亲很喜欢写故事。 My father likes to very much. 63. 请把这些肉切碎。 Please the meat! 64. 如果你来,我会和你一起去。 If you , I go with you. 65. 你想要多大碗的? would you like? 八、书面表达。(10分) 根据下面的提示写一篇短文介绍你上周末的厦门之旅。 1. 上周末和父母一起去了厦门,住在叔叔家。 2. 星期六叔叔带我到厦门大学(university)参观。 3. 星期天参观了鼓浪屿(Gulangyu)和南普陀庙(South Putuo Temple). 4. 周末玩的很愉快。 初一英语下册期末试卷答案 一、单项选择:(15分) 评分标准:每小题1分,共15分。 1-5. CBBDA 二、完形填空 (10分) 评分标准:每小题1分,共10分 16-20. BCBAC 21-25. BAACB 三、阅读理解(30分) 评分标准:每小题2分,A、B、C三篇共30分 26-30. BCBDA 31-35. DCBCB 36-40 CBDBA 四、交际运用:(5分) 评分标准:要求写序号,写句子不给分,每小题1分,共5分。 41-45 DCEBA 五、句型转换:( 10分) 评分标准:按空给分,每空1分,每小题2分,共10分。 46、Don’t wear 47、How was 48、didn’t do 49、What’s like 50、between and 六、词语运用: (10分) (A)根据句意及首字母提示完成单词。 评分标准:每小题1分,共5分 51、Ninety 52、rules 53、library 54、expensive 55、musician (B)用所给词的适当形式填空。 评分标准:每小题1分,共5分 56、skating 57、candies 58、took 59、is using 60、to do 七、 完成句子(10分) 评分标准:按空给分,每空1分,每小题2分,共10分。 61、on time 62、write stories 63. cut up 64、come, will 65、What size 八、 书面表达:(10分) 评分标准: 1、作文包含所要求的内容,并有适当的发挥,句子通顺,条理清楚, 没有语法和单词错误可以给予10分。 2、作文包含所要求的内容,句子通顺,存在单词和语法错误,可以给7--9分。 3、内容不完整,句子较通顺,老师酌情给分。猜你喜欢: 1. 七年级英语期末备考卷及答案 2. 新人教版七年级英语下册期末测试卷 3. 七年级英语期末考试卷附答案 4. 2017七年级英语下册期末试卷及答案 5. 七年级下册英语期末考试卷及答案

初一的同学们,期末考试就要来了,英语还没复习?那就先来做份往年的英语试卷吧。下面由我为大家提供关于初一期末考英语试卷及参考答案,希望对大家有帮助!

二、单项填空 (共15小题;每小题1分,满分15分)

请认真阅读下面各题,然后从A、B、C、D四个选项中,选出可以填入空白处的

最佳选项,并在答题卡上将该项涂黑。

16. — You look happy, Millie. Anything special?

— Yes, Mum. We have ______ “Festival Week” at school. We all have ______ good time.

A. the; a B. a; the C. a; a D. the; /

17. — Daniel is only five years old, but he can draw very well.

— How ______ he is!

A. quick B. careful C. interesting D. clever

18. — That’s all for this lesson. Do you have any ______?

— No, thank you , Mr Li. We can understand very well with your help.

A. ideas B. questions C. plans D. activities

19. — Are you the only child in your family?

— Yes. I have no brother ______ no sister.

A. or B. but C. and D. so

20. — Do you like chocolate?

— No, I don’t, but I ______ have a little to give me some energy.

A. sometimes B. often C. usually D. always

21. — Show me your presents for your parents, will you?

— Look! The bag is for ______. And the sunglasses are ______.

A. Mum; Dad B. Mum ’s; Dad’s C. Mum’s; Dad D. Mum; Dad’s

22. — ______ are the students?

— They are reading in the library.

A. What B. Who C. Where D. How

23. — Here are all kinds of juice. ______ do you want?

— 10 bottles of apple juice and 10 orange juice. ______ are they?

A. How many; How many B. How much; How much

C. How much; How many D. How many; How much

24. — I watched a very ______ cartoon last night. The mouse is playing happily with the cat.

— Oh, that is really great ______. I want to watch it, too.

A. fun; fun B. funny; funny C. funny; fun D. fun; funny

25. — Look at the boy. He is too young to ______ all these books.

— Oh, let’s go and help him.

A. take B. bring C. get D. carry

26. — You ______ lessons this Thursday, right?

— No, we had a picnic in the park.

A. don’t have B. haven’t C. didn’t have D. hadn’t

27. — Why were you late for school this morning?

— My mother forgot to ______.

A. pick me up B. wake me up C. dress me up D. get me up

28. — Is that Jack over there? It seems he ______ the cow.

— It can’t be him. I saw him watching TV when I walked past his house.

A. is milking B. milks C. will milk D. milked

29. — Can I have ______ juice, Mum?

— Sorry, what about ______ milk?

A. some; any B. any; some C. some; some D. any; any

30. — ______ chess with me?

— Yes, I’d love to.

A. Do you like playing B. Do you like to play

C. Would you please play D. Would you like to play

三、完形填空 (共10小题;每小题1分,满分10分)

请先通读下面的短文,掌握其大意,然后从每小题所给的A、B、C、D四个选项中选出最佳选项,并在答题卡上将该项涂黑。

Father came to Jack’s bed and sat down beside him.

“ 31 was your day?” a sked Father.

“It wasn’t a very good day for me,” Jack answered. “I had a 32 with Tom. The teacher sent a note home about me. And I talked back to Mother.”

“Yes, part of the day was 33 ,” his father said. “But tell me about the best thing that you had today.”

Jack thought for a minute, and then smiled. “After school I went fishing alone,” he said. “I didn’t think I would have any 34 . But I did. I caught a fish!”

“I’m glad you enjoyed youself,” Jack’s father said. “Think about it until you go to 35 . Now I’m going to turn 36 the light. Have happy dreams.”

Every night, Jack answers the same question. “What was the best thing that 37 to you today?” Every night, Jack thinks about the best part of his day. This gives him many happy dreams.

Sometimes he has to think 38 to find the best thing. Sometimes it is a very 39 thing. But he always finds one good thing to think about. Jack’s father 40 him to end his days in a wonderful way.

31. A. What B. How C. Where D. When

32. A. talk B. lesson C. game D. fight

33. A. good B. exciting C. bad D. sad

34. A. luck B. energy C. time D. plan

35. A. rest B. sleep C. dream D. bed

36. A. up B. down C. on D. off

37. A. put B. happened C. lent D. meant

38. A. hard B. together C. happily D. first

39. A. interesting B. special C. small D. different

40. A. asked B. taught C. told D. called

四、阅读理解 (共10小题;每小题2分,满分20分)

请认真阅读下列短文,然后从每小题所给的A、B、C、D选项四个选项中,选出最佳选项,并在答题卡上将该项涂黑。

A

Oh, the rain! How Sara hates the rain! She cannot go outside to meet her friends for a game. He r mother says to her, “You may catch a cold if you play in the rain.” But Sara shouts, “It’s raining, but it’s very hot outside!” Her mother still doesn’t let her in the rain.

Sara goes up to her room and lies down in front of her window. In fact she has many things to do in her room. There is a TV, some books and toys. But she doesn’t like any of them.

Sara sits by the window. She does everything to make the rain go away. She opens the window so the rain can hear her. She wishes the rain away but that doesn’t work. She asks it to go away but that still doesn’t work. She stands up and shouts at the rain but that still doesn’t work.

“Well, if you’re going to be here and ruin my day, you can find something interesting for me to do.” She shouts and again she gets no answer.

So she sits back down on the window. It is still raining. “Fine,” she thinks, “I c an go swimming, ride my bike and play with my friends when the rain stops.” She thinks about her ideas again and again. How she wishes the sun to come out! Soon she falls asleep on the window. (211)

41. What’s the weather like that day?

A. Rainy and hot. B. Rainy and cold. C. Cold and sunny. D. Hot and sunny.

42. What does the word “ruin” mean in Chinese?

A. 喜欢 B. 改变 C. 毁掉 D. 度过

43. From the passage, we know ______.

A. the rain stops and the sun comes out

B. Sara finally falls asleep on the window

C. Sara is interested in playing in the rain

D. Sara’s mother doesn’t want her to play with her friends

B

The street food stands (摊) are very popular in Korea. The stand owners give the names of their food stands according to (根据) the food they sell (出售): Dukbokgi and Odeng.

Dukbokgi is a spicy hot food made of rice cakes and some vegetables. These rice cakes are finger-s haped. They are white, and they are about ten centimeters long. They are mixed with sliced vegetables in a pan with a spicy red dressing. Fifteen minutes later, they are ready to eat.

Odeng is made of ground fish. It comes in many shapes: some are square, others are round, and still others are stick style. You can buy big or small ones. At these food stands, you can also eat Odeng. It is boiled (煮) in a pot. It is served on a long stick so you can eat it easily.

The food at these stands is not expensive. Each Dukbokgi and Odeng is about twenty to fifty cents, depending on the owner.

Each food stand has its own recipes (配方) for its food; for example, some like to add sugar or other special ingredients (配料) in the food. Usually, there is just one owner, but if the food stand is big, there may be two or three owners.

When I was a primary school student, I used to stop at street food stands every day with other girls after school to eat the street food.

44. The owners take the names of their stands from ______.

A. the food they sell B. their recipes

C. the special ingredients D. the shape of the food

45. What are the ingredients of the Dukbokgi?

A. Rice and fish. B. Rice cakes and sugar.

C. Fish and vegetables. D. Rice cakes and vegetables.

46. Which of the following is NOT true?

A. Odeng can be made into all kinds of shapes.

B. Stand owners add different ingredients into street food.

C. Street food is popular in Korea because it’s made easily.

D. The writer liked street food when she was in primary school.

47. What’s the best title for the passage?

A. My Favourite Food B. Street Food Stands in Korea

C. The Most Popular Food in Korea D. How to Make Street Food

C

Briana, a student at John Fenwick School in Salem County, US, has a lot of free time. The 13-year-old girl used to hang out (闲逛) on the streets after school. “I know it wasn’t good, but I really had nothing else to do,” Briana said.

Briana was not alone. Many kids in her city had too much free time and nothing to do. So four school districts in Salem began a program (项目) called Big Brothers / Big Sisters. It helps students do something useful after school. Big Brothers / Big Sisters asks mentors (导师) to help students in grades 6 to 8 build healthy relationships (关系) and take part in some activities.

The “Littles” and the “Bigs” are nicknames for students and mentors. Most mentors are teachers. The “Bigs” and “Littles” usually meet once a week. They play games, share stories and go on trips.

The program has already helped many Salem students.

Briana’s mentor is a school headmaster, Syeda Woods. Woods took Briana ice-skating, to pizza parties and for a visit to Longwood Gardens in Pennsylvania. “ When I got out, I saw that Salem is a very small place,” Briana said. “The program helps me experience (体验) the outside world and see many new things.”

Kathy Jennings, 13, said she was very quiet but now she likes talking with others and making new friends.

“In the program, I see my mentor as a big sister, not a teacher. I can tell her anything,” Jennings said. “And she has taught me a lot about making good decisions (决定). I think it will make a big difference in my life.”

48. The program is to help students ______.

A. make good use of after-school time B. get high grades

C. to be kind to their brothers and sisters D. take part in school activities

49. What does Briana think of the program?

A. It helps her to know her school teachers well. B. It brings her many new friends.

C. It helps her understand the world better. D. It takes a lot of her free time.

50. What is Kathy like now?

A. She is quiet and different. B. She is like a big sister.

C. She is good at making decisions. D. She is much more open.

五、任务型阅读 (共5小题;每小题1分,满分5分)

阅读下列家谱图,完成各小题。每空只写一词。

51. White is Mary’s ▲ name.

52. ▲ is Ann’s aunt.

53. John is ▲ uncle.

54. Ann is Mary’s ▲ .

55. Bruce and Allen are Mary’s ▲ .

六、词汇检测 (共10小题;每小题1分,满分10分)

根据下列句子所给首字母或汉语注释以及上下文,在答题卡标有题号的横线

上,写出空缺处各单词的正确形式。每空只写一词。

56. The food t ▲ nice, so he wants to eat more.

57. Amy is c ▲ her birthday with her friends at home.

58. You should act like ▲ (绅士), my boys.

59. The present ▲ (花费) me only two y uan.

60. The little boy can count the ▲ (数) from one to one hundred.

61. — ▲ of the subjects do you like best?

— English. I hope to study in the US.

62. — What is the dress made of?

— ▲ . If feels so soft and smooth.

63. — Are you ready to start?

— Just give me ▲ five minutes.

64. — The children in that area can’t go to school.

— What ▲ children! Let’s help them.

65. — Can I have more sweets, Mum?

— No, ▲ they are bad for your teeth.

七、完成句子 (共5小题;每空分,满分5分)

根据要求完成句子,并将答案写在答题卡标有题号的横线上。

66. Our library has all kinds of books. (改为同义句)

▲ ▲ all kinds of books in our library.

67. He looks strong. (对划线部分提问)

▲ does he look ▲ ?

68. She is my hero. (改为复数句子)

▲ my ▲ .

69. If you eat too many sweets, you’ll be fat. (改为并列句)

▲ eat too many sweets, ▲ you’ll be fat.

70. He often lies on the bed listening to the music. (用now改写句子)

He ▲ ▲ on the bed listening to the music now.

八、句子翻译 (共5小题;每小题2分,满分10分)

将下列句子译成英语,并将所译句子写在答题卡标有题号的横线上。

71. 我的梦想和我同学们的不同。

72. 我有必要改变我的生活方式来保持健康。

73. 我每周三晚上练习游泳。

74. 我享受周末与家人团聚在一起。

75. 我希望有早一日我的梦想成真。

九、书面表达 (共l题;满分10分)

76. 为了迎接新年,你班在举行时装表演。请你根据提供的人物与图片,并以主持人的身份,用英语对此进行现场报道。

人物 服饰搭配 服饰颜色及材质搭配

合理地自由搭配

合理地自由搭配

合理地自由搭配

注意:1.词数70左右。短文的开头、结尾和连接部分已为你写好,不计入总词数;

2.衣服的颜色及材质可根据自己的喜好进行搭配,注意搭配要得当;

3.短文须包括所有要点,不要逐词翻译,可适当发挥,使短文连贯、通顺。

Good afternoon, everyone! Welcome to our fashion show. I’m Kitty from Class1, Grade 7. Today we are going to show you all kinds of clothes.

Here comes Daniel. ▲

Now Sandy and Simon are coming. ▲

Look! Here comes Amy. ▲

They all look modern and smart. That’s all for today’s show. Thanks for coming.

初一英语月考试卷

在新的学期马上又将迎来月考考试,这是对大家一个节点的检测,接下来是我为大家带来的关于外研版 七年级英语 下册月考试卷,希望会对大家有所帮助。

外研版七年级英语下册月考试卷:

一.根据句意及首字母提示,补全句中所缺单词(5分)

sister likes playing the p_______.

are five h_______students playing on the playground.

you like to fly a k________with me?

Yes ,I’d love to .

hobby is to c_______stamps .What about yours?

’t w_____,boy ,I can help you.

二.用所给词的适当形式填空(10分)

you ______(cook) some chicken for dinner,mum?

is my book .Where is ______(you)?

(who) tape is this .Tony?

day ________(thousand) of tourists come to Beijing to visit the Great Wall.

would like ________(join) an English club.

___________________(have) a piano lesson next Sunday afternoon

should get on________(good)with our classmates.

! who ________(read) English?

enjoys ________(listen) to music in her free time.

is looking forward to________(visit)Shanghai next month.

三.选择填空(20分)

( )1. ________bike is this?

I think it’s peter’s

C. whose D. what

( ) Ming is good at math .He often helps_____with it

C. us

( ) this ______ruler ?

over there.

Her B. her ;Hers C. hers;Hers D. hers ;Her

( )

He can ride a horse

can he do? is he?

C. Can he ride a horse is he?

( ) morning ,I am going to do my homework and clean my room

B. In C. At D. About

( ) friend Jane _______a test next is going over lessons at home,now

having C. is going to have having

( ) interesting story makes me________________.

A happy C. angry

( ) Lucy going to the cinema on Sunday?

will have a piano lesson.

is B. No,she isn’t C. Yes ,she will won’t

( ) you going with?

With my parents

else else C. when else else

( ) teaches ______painting?

teach________.

;mine ;my C. you ; me ;myself

( ) hope ______the guitar

to play can play C. playing D. to playing

( ) promises_______his mother _______cleaning at home.

help , to do B. to help ,doing C. help ,todo D. help ,do

( ) can play _____guitar in your school?

B. the C. an /

( )’s father enjoy_____English songs?

to B. to listen C. listening to D. to listen to

( ) you like to go out for a walk with me ?

_____, but I must finish my homework first.

course not ’s all right C. I’d love to D. Yes, I do

( ) you

-----_____________

’t say that ’s right C. That’s all right D. It’s a welcome

( ) you ______plant trees on March 12th?

with C. ready in D. ready to

( ) want to ________the swimming club.

C. take part in

( ) are your plans______the weekend?

C. witb

( ) often _______my homework at home.

A forget C. lose D look

四.汉译英(15分)

1.我们应该学会与别人友好相处。

We should learn to ___ _____ ______ _____others.

2.贝蒂每天上学都很忙。

Betty goes to school____ ______ ______every day.

3.他们正盼望着明天的野餐。

They ______ ____ _____ ______the picnic tomorrow.

4.我正在找我的手表。

_______ _______ ______my watch.

5.欢迎回到北京

______ _____ _______Beijing

6.从现在开始,你应该学更多英语。

______ _____ _____,you should learn more English.

7每年的十月份,都会有成千上万的游客去北京观光。

Thousands of visitors_______ ___________in Beijing in October every year.

8.这个星期天你们打算去打 篮球 吗?

____you _____ ______play basketball this Sunday?

9.我汉语说得不太好。

I can’t ____Chinese very _______

10.我们将会在上海玩得很愉快

We are going to _____ _____in Shanghai

五.句型转换(20分)

is Mary’s pen(划线部分提问)

______ ______is this?

at this nice watch, Is it your watch?(改为同义句)

Look at this nice watch .Is it ________?

you ride a bike ?(作否定回答)

_____, ______ ______

wants to join the Chinese Club (改为同义句)

He ______ _______to join the Chinese Club .

is going to play football with his friends this afternoon(划线部分提问)

______ _____he ____ _____ _____with his friends this afternoon?

can swim(划线部分提问)\

_____ _____he ________?

have a picnic on Friday(用next Friday.改写)

We ____ ____ _____ _____a picnic next Friday.

六.补全对话(10分)

(A)选择正确的选项补全对话(有两个多余的选项)

Betty :Hello ,Tony!Where are you going this Sunday?

Tony:(1)__________________

Betty :Ithink it’s not very interesting .Why are you going there?

Tony: (2)_________________-

Betty :Wow,I think you are a good brother.

Tony:Of course!(3)_______________

Betty :I’m going to China National Film Museum(中国电影博物馆)

Tony: (4)_______________

Betty :Yes, I can .There are lots of films about children.

Tony:Wow !That’s great !(5)_____________

Betty :I’m going to take the bus .

Tony:Oh,I know .You can take the bus

Betty :Yes,you are right.

are you going at the weekend?

is that?

C .I’m going to Beijing Zoo with my sister

are you going?

you watch a film there?

,that’s a great idea!

my sister wants to go there.

(B)

A:What are you__1___________?

B:We are__2_________________the house.

A:The spring festival is coming .Are you getting_3_____________for it?

B:Yes ,we are.

A:What’s your mother doing?

B:She’s _4________dinner for us in the kitchen(厨房)

A:Is your father helping you?

B:No, he isn’t .He is at 5_________.

七.完型填空(10分)

Do you know Eskimos(爱斯基摩人)?Let me tell you something __1___their life .

The Eskimos live near the North Pole (北极).There are only two seasons there: winter and summer .There is no spring ___2__autumn there .In winter ,nights are long .You can’t __3__the sun for more than two months ,even at summer ,days are more than two months ,the sun never __4__and there is no night.

Eskimos have __5__clothes .They make their cloths from the skins(皮肤)of animals .From skins they make coats,hats and the North Pole trees can’t grow ,because it is Eskimos have to make their houses from skins ,stones or snow .When they go out in a storm (风暴)and can’t get back__8__,they make houses of snow .They leave these snow houses when the storm is ____9__.

Life is ___10___for the Eskimos ,but they still like to live there.

( ) D. with

( ) D. with

( ) at

( ) .comes out B. comes over down up

( ) C. cotton

( ) B. drinks C. medicine

( ) cold B. too hot cold or hot so cold

( ) home B. home C. to home D. at home

( ) D. on

( )

八.阅读理解(20分)

(A)

Mrs Brown goes to see her son in London .Her son works in a music club there .Mrs Brown doesn’t know London very she can’t find her way.

She sees a man at a bus wants to ask the man the way.”Excuse me!Can you help me,please? Which bus goes to Miller’s Shop?”Mrs Brown asks.

The man is very friendly .He smiles(微笑). But he can’t speak English .He can speak French .He puts his hand into his coat and takes out a small book .He opens it and reads something on it “I am can’t speak English.”

( ) Brown goes to ______in London.

in a club the way her son the man

( ) Brown ‘s son works in_______

big school ’s shop C a music club bus stop

( )’s “bus stop “ in Chinese ? A.车库 B. 加油站 C交通灯 D.车站

( ) Brown can’t find her way because_________

is old is new in London

C she doesn’t know London well D. she can’t see

( ) is right?

man at the bus stop is an Englishman.

man can’t help Mrs B rown

C Mrs Brown can’t speak English

Brown borrows the book from the man

(B)

Adam is goes to a boarding school(寄宿学校).He stays at school five days a week.

He is in the baseball club and music club. He likes baseball very much and he is good at singing and music instruments(乐器).He can play the violin ,piano and guitar parents ,Mr and Mrs Smith ,are music teachers .They teach in another school .They like music very much and they are successful in often sing at singing shows on Tv .Adam’s sister,Sarah,is can sing a lot of songs .She can dance well ,too. She says she wants to be a musician and a famous dancer.

( ) are______people in Adam’s family.

( ) is a _____-

D. doctor

( )3Acording to the passage ,what can Adam do?

He can _____.

chess B. play the guitar football D. play table tennis

( ) Smith teaches____--- xK b1 . Co m

B. . C. math D. English

( ) is good at ________.

B. teaching

(C)

Do you like sports? Are you getting enough exercise? Well, if you’re looking for a way to exercise, try walking.

●Waking is easy. There are no special instructions, and there are no rules. You just walk-left, right, left, right. See? It’s easy.

●Walking is cheap. Don’t spend money on expensive clothes and equipment(器材). All you need is a pair of comfortable shoes.

●Walking is fun, and it gives you time for yourself. Go with a friend. Walk and talk. Do something you enjoy. Listen to music, think about life, relax and so on.

●Walking is good for your health. It’s good for your mind and body. You feel energetic after walking a lot. And now science is proving that walking outdoors is the best exercise.

●Walking is good in all kinds of weather. So it’s raining? Don’t worry, take an umbrella. When it’s sunny, use sunglasses. If it’s hot, take some water with you. When it’s cold, you always feel warm. And walking in the snow is of great fun!

Are you willing to walk? Don’t wait! Just do it!

根据短文内容, 判断 句子 正(T)、误(F)。

( )1. Here are four reasons why walking is a great idea.

( ) must follow some rules while you are walking.

( )3. If you walk, you needn’t buy expensive clothes.

( )4. Walking is boring because you can’t walk on rainy days.

( ) passage tells us walking is a good way to keep healthy.

(D)

Jim’s timetable

Sunday Monday Tuesday Wednesday

Go to the zoo

with his pet Cody Do somereading Swim in the afternoon Send an email to Tom

Thursday Friday Saturday

Learn to drive a tractor from Uncle Wang Watch Tv in the evening Help Mum clean the house

in the afternoon

1. what’s Jim going to do on Monday?

2. Is Jim going to the zoo on Sunday with his pet Cody?

3. Is Jim going to help his father clean the house?

4. When is Jim going to send an email to Tom?

5. What is Jim going to do at Uncle Wang’s?

九.书面表达(10分)

请根据下面表格中的内容描述一下John下周末的活动。要求包括以下内容,可适当发挥。

John’s plans Saturday morning Saturday afternoon Saturday evening

do his homework clean the room go shopping

Sunday morning Sunday afternoon Sunday evening

七年级英语 即将迎来第三次月考考试,同学们要准备哪些月考试题来复习才会事半功倍呢?下面是我为大家带来的关于七年级英语第三次月考试题,希望会给大家带来帮助。 七年级英语第三次月考试题: 一、单项选择(40分) ( )1. She likes soccer after class. A. played ( ) of medium build and curly hair . A. has, has B. is ,is ( )3. I usually _____ one hour ______ soccer with my friends on Sunday afternoon. A. spend, playing B. spend, play C. take, on D. take, to play ( )4. You must remember _______ the window before you leave. A. closing B. close C. to close D. closes ( )5. --- ? ---He has big eyes and small nose . old is he ? does he look like ? does he do ? is he ? ( )6. It’s a day. Let’s play at home. A. rainy B. sunny C. rain D. snow ( )7. There some fish in the bowl and some eggs on the table. A. is B. has C. are D. have ( )8. I’d like some and . A. porridges, vegetable , tomatoes C. potatos, bananas D. French fries, orange juices ( )9 Andrew enjoys ___________ very much. A. play football B. playing football C. playing the football D. to play football ( )10. Julie would like TV. A. watch B. watches C. watching D. to watch ( ) of shoes do you wear? --- Size 7. A. What B. What size C. What kind D. What color ( )12. The _______ noodles are very popular in this city. A. muttons and eggs B. mutton and carrots C. mutton and carrot D. muttons and carrot ( )13. ---Would you like _____ tea? ---Yes, _____. A. some, please B. any, please C. any, I will D. some, I would ( )14. ---What about some fish? --- ___________, I’m full(吃饱了). A. Yes, please B. Yes, I do. C. No, thanks D. I’d love to ( )15. What size bowl of dumplings would you like, small, medium ____ large? A. and B. with C. or D. But ( ) 16. My homework is difficult. A. little B. a little C. few D. a few ( ) 17. --- Lucy practicing playing the piano right now? ---Yes, she . A. Do; do B. Does; does C. Is; is D. Is; does ( ) 18. --- is your vacation? --- Oh, it is great! A. Why B. How C. When D. Where ( ) 19. ---What about to the supermarket with me? ---That’s a good idea. A. going B. to go C. went D. goes ( ) 20. Sarah a funny story now. A. is reading B. looking at C. is read D. are watching ( ) 21. Look! The young man is coming. He a pair of cool sunglasses. A. puts on B. is puting on C. is wearing D. wear ( ) 22. Mr Wang ___________in this school. A. don’t teach B. not teach C. doesn’t teach D. doesn’t teaches ( ) 23. Do you know ? A. How does your father look like B. What your father looks like C. Who does your father look like D. Where your father looks like ( ) 24. I think I know her. She has _____ hair. A. short blonde curly B. short curly blonde C. blonde short curly D. curly blonde short ( ) 25. Michel Jackson, the rock(摇滚) singer long curly black hair. A. has B. have C. with D. of ( ) 26. there on time, he gets up early. A. To get B. Getting to C. Go D. Get ( ) 27. please tell Jim watch too much TV after dinner. A. doesn’t B. don’t C. isn’t D. not to ( ) zoo is this weekend. A. lucky B. busy C. easy D. difficult ( ) 29. Each of us an English name. A. have B. has C. are D. does ( ) 30. Molly makes a wish and the candles. A. blows out B. put out C. cuts down D. cuts up ( ) are tomatoes on the table, but there isn’t mutton on it. A. some, some B. some, any C. any , any D. any, some. ( ) 32. He doesn’t like coffee milk, but he likes coke orange juice. A. and, and B. and , or C. or, and D. or, or ( ) 33. There are some differences _______ me _______ my sister. A. from; to ; from C. from; and ; and ( ) 34. ---Where is Jimmy now? ---Oh, I think he _________ his car in the garden. A. washes B. doesn’t wash C. is wash D. is washing ( ) 35. There are two _________ reading English at the English Corner(英语角). A. student B. mans C. students D. peoples ( ) 36. Don’t pay for the coffee. It’s . A. good B. tasty C. boring D. free ( ) 37. --- _______ the banks? ---They are across from the hotel. A. Where is B. Are there C. Is there D. Where are ( ) 38. ---How can I get to the post office? ---Walk this road, turn left the third crossing. It’s the right. A. down; at; on B. on; along; in C. through; in; on D. along; on; at ( ) 39. I am not sure . A. always B. still C. never D. yet ( ) 40. Mike likes to watch the birds _________ in the sky. A. flys B. flying C. flies D. fly 二、完型填空(10分) People in different like to eat different food. kind of food do people would like For example , fried potatoes chips are very . Sometimes people them at home, but they usually go to a fast food restaurant them. They can them in the restaurant bring them away at home. Sometimes, they like to eat them in the park. ( ) 41. A. cities B. countries C. towns D. homes ( ) 42. A. where B. when C. what D. who ( ) 43. A. eat B. to eat C. eating D. eats ( ) 44. A. in a UK B. in an UK C. in the UK D. in UK ( ) 45. A. popular B. large C. small D. cheap ( ) 46. A. cooking B. cooks C. are cooking D. cook ( ) 47. A. buy B. buys C. buying D. to buy ( ) 48. A. has B. have C. to have D. having ( ) 49. A. and B. but C. or D. so ( ) 50. A. too B. also C. either D. and 三、阅读理解(25分) A I`m Tony. My best friends are Frank and Cindy. We often do many things together. Frank lives next to my home and we are in the same class. He is thirteen years old and tall and medium build. He has curly hair and blue eyes. He is good-looking and clever. He is good at math and often helps me with my homework. He likes wearing black pants and yellow T-shirts. Cindy doesn`t go to my school. She is eleven years old. She is thin and medium height. She has curly blonde hair and her eyes are brown. She is good-looking, too, but a little bit shy. She is good at playing the guitar. Her facorite subject at school is music. She often teaches Frank and me to play the guitar after class. The three of us have great fun together, and we help each other and sometimes play video games at my house. 51. eyes are blue. ’s ’s ’s 52. good at math . is is is am 53. is good-looking ,but a little bit shy . friend do they play video games ? school Frank’s house Cindy’s house Tony’s house things do the three children do together ? They . baseball video games the guitar B&C B You’re welcome, ladies and gentlemen. Everything in our restaurant is cheap and ’s have a look. For breakfast, you can eat porridge, eggs and noodles. For lunch and dinner, you can have rice, dumplings, different kinds of meat and vegetables. There are also some drinks for you. Come and have a meal, please. It’s a good place to enjoy yourself. breakfast: porridge($), egg($), noodles($$). lunch and dinner: rice ($), dumplings ($), cabbage ($), carrots ($), potatoes($), chicken ($), fish ($), mutton ($);green tea ($), iced tea ($), orange juice ($). business hours: . 6:30 . ; . 7:00 .. ( )56. What can you eat for breakfast in the restaurant _________. A. dumplings. B. beef noodles. C. ice cream. D. french fries. ( ) 57. You can have a meal in the restaurant ________. A. at 6:40 on Saturday morning B. at 9:45 every evening C. at 11:30 every morning D. at 11:30 on Sunday night ( ) 58. If you have rice, cabbage, and green tea for lunch, you need to pay ___________. A. $ 8 B. $9 C. $10 D. $11 ( ) 59. Mr black goes to the restaurant with $ 22 for dinner. He can order rice, potatoes, chicken and _________. A. iced tea B. orange juice C. green tea D. an egg ( ) 60. What does the speaker probably(可能) do ? A. a reporter. B. an artist. C. a waiter. D. an actor. C My sister Sharon and I look very different. She’s tall, and I’m short. She has short curly dark (黑暗的,深色的) hair, and I have long straight blond hair. We both have dark eyes, but Sharon wears glasses, and I don’t. And she always wears large earrings (耳环). I don’t wear earring. Sharon and I do many things together (一起). We watch movies together, and we often go shopping. We play tennis together every Saturday. Often, people don’t know that Sharon and I are sisters, because we look so different. But we are also good friends. ( ) 61. What does Sharon look like? She . A. is tall and has dark hair B. doesn’t wear earrings C. has long straight blond hair D. is short and dark hair ( ) 62. The sisters don’t together. A. play the piano B. play tennis C. go shopping D. watch movies ( ) 63. The two sisters both_ . A. wear glasses B. have blond hair C. like earrings D. have dark eyes ( ) 64. What does the word “ different” mean in Chinese? A.一样的 B. 困难的 C. 不一样的 D. 友善的 ( ) 65. Which is the best title (标题) for this reading? A. Playing Tennis B. My Hobbies. C. Two sisters D. Meeting New Fiends D It’s Sunday morning. There are many people in the park, some boys are playing basketball. There are some girls under a big tree. They’re singing and dancing. What are those women doing? They are drinking tea. Look at the woman in a blue coat. Who’s she? She’s my mother. She is talking to Edward. Edward is her student. He is a good student. He studies hard. He’s good at all the subjects. And he’s friendly to all his teachers and his friends. ( )66. The boys are ______. A. singing B. playing basketball C. dancing D. drinking ( )67. My mother is ______. A. a teacher B. an office C. a doctor D. a woman ( )68. Edward is in ______. A. his home B. the retaurant C. the zoo D. the park ( )69. Who’s under a big tree? A. Some people. B. Some women. C. Some girls. D. Some boys. ( ) color is my mother’s coat? It’s ______. A. blue B. white C. red D. yellow E The final exam comes with June. When the exam ends(结束), the summer vacation begins. Boys and girls have about two months to relax. The summer vacation is the best time of the year for most children. The weather is usually fine. They can swim, go to summer camp or visit other places with their parents. Of course, the beaches are good places for relaxing. Some children are lucky to live near the beach. They can play in the water. But for the children far from the sea, they go to the beaches for one or two weeks with their parents. Why do children like spending their summer vacation on the beaches? It is because they like the sand(沙), the sun, the cool wind and the sea water. There are lots of new things to see, nice things to eat, and exciting things to do. ( )71. School children usually have exams _____. A. in June B. after June C. before June D. in July ( )72. The summer vacation is about ______. A. two weeks B. two months C. three months D. two days ( )73. Why do children like summer vacation? A. because the weather is fine. B. because it is long. C. because the weather is fine and they can have lots of activities(活动). D. because they can have exams. ( )74. Children far from the beach can enjoy the sea ______. A. in the evening B. very often C. for two months D. for one or two weeks ( )75. Which of the following is not right? like going to the beaches only because they can see lots of new things. B. Children like summer vacation best. C. Children can swim and enjoy the sand, wind, and water on the beaches. D. Children like to spend their summer vacations on beaches. 四、综合语言知识(选出可以替换划线部分且意思一致的一项,共5分) 76. Why don’t you come and have a look? A. Why don’t B. Why not C. Why about D. What don’t 77. I spent five yuan on this pen. = I spent five yuan buying this pen. A. by B. in C. for D. on 78. He is of medium build. A. not tall or short B. thin or heavy C. of medium height D. not thin or heavy 79. We often take a taxi to go there. A. go there by taxi B. by taxi to there C. take taxi go there D. go there to take a taxi 80. There isn’t any water in the bottle. A. is not B. is never C. is no D. is any 五、完成 句子 (每空一词,每词一分,共12分) 81. There are three ______________ (特色菜) for today. 82. Turn left at the first ___________ (十字路口) and you can see a ___________ (餐馆) on your ___________ (右边) . 83. Eason Chan is a ___________ (受欢迎的) __________ (歌手) in our country. 84. His sister is of medium ___________ (身高) and has ___________ (直的) hair. 85. There is a _________ (图书馆) in our school. 86. Can you ___________ (描写) your friend for me? 87. The birthday foods bring good ___________ (运气) to the birthday ___________ (人). 六、任务型阅读。(8分) Tony is six years old. He lives with his parents and grandparents. This afternoon, Tony’s teacher, Mr. King, calls. Little Tony answers the phone. “Can I speak to your father?” Mr. King Asks. Tony says, “Sorry,Mr. father is busy.” Mr. King asks, “what is he doing?” “Talking to a fireman(消防员), ” the boy answers. “Then can I speak to your mother, boy?” Mr. King asks. “Sorry, but she is busy talking to a policeman, ” says Tony. “What about your grandparents? Are they busy, too?” “Yes, they are.” “Can I speak to any adults(成年人)in your family?” asks the man.“No, you can’t. They can’t find me and they are all busy looking for(寻找)me,” says the little boy. 根据短文内容完成任务,回答下面的问题。 88. What is Tony’s father busy doing? _________________________________________________ 89. _________ is busy talking to a policeman. A. Mr. King B. Tony C. The policeman D. Tony’s mother 90. 把划线句子翻译成汉语___________________________________ 91. 划线的they指__________________________________________(填英语) 七、补全对话(根据已给出的内容,补全对话。每题2分,共10分。) A:Good afternoon. May I 92 _____ _____ _____ _ _____ _____ _____ ? . 93 _____ _____ _____ __ vegetables in the beef soup? A:Yes. There are some tomatoes. . B: OK. We’d like one bowl of beef soup. A. Is her hair long? B. Is she thin? C. What does she look like? D. What color is her hair? E. Does she have two big eyes? A: Sure. 94 _____ _____ _____ _____ _____ _______ _____ _______? B: Large, please. A: We’d also like gongbao chicken and some mapo tofu with rice. B: OK. 95 _____ _____ _____ _____ _____ __________ _, one gongbao chicken and one mapo tofu with rice. A:Yes, 96 _____ _____ _____ ___ _____ ______ . 八、书面表达(10分) 写一篇 英语 作文 介绍一位好友。(建议:为避免透露姓名,女名用Lisa,男名用Mike。)

做好七年级每份英语月考试卷,就是对七年级英语知识的一个学习和巩固的过程。以下是我给你推荐的七年级英语第二学期月考试卷及参考答案,希望对你有帮助! 七年级英语第二学期月考试卷 一.听力理解 A根据所听句子的内容和所提的问题,选择符合题意的回答问题,并将答案写在题前的括号内。每小题听一遍。(共5小题,每小题1分,共5分) ( ) is Tom doing? ( ) can Peter do? ( ) does Ann go to school? ( ) can’t we do here? B.听对话(共10小题,每小题1分,共10分) 根据所听对话的内容和所提的问题,在各小题所给的三个选项中选出一个最佳选项,并将答案写在题前的括号内。每段对话听两遍。 听第一段对话,回答第6小题。 ( ) does the boy usually do at 6:30 runs. gets up. has breakfast. 听第二段对话,回答第7小题。 ( ) time does Ricky have dinner? six in the morning. six in the evening. eight in the evening. 听第三段对话,回答第8小题。 ( ) does the boy want to know? girl’s phone number. girl’s e-mail address. girl’s school address. 听第四段对话,回答第9小题。 ( ) does the girl think of the tigers? ’re cute. ’re funny. ’re scary. 听第五段对话,回答第10小题。 ( ) does Jenny usually have lunch? home. her school. her grandparents’ home. 听第六段对话,回答第11~12小题。 ( ) may it be? the morning. the afternoon. the evening. ( ) do they want to go? zoo. school. park. 听第七段对话,回答第13~15小题。 ( ) club does the boy want to join? swimming club. English club. chess club. ( )’s the boy’s name? Black. Smith. Smith. ( ) old is the boy? years old. years old. years old. C.听短文(共5小题,每小题1分,共5分) 根据所听短文的内容,在各小题所给的三个选项中选出一个能回答问题的最佳选项,并将答案写在题前的括号内。短文听两遍。 ( ) do koalas like staying? the grass. the tree. the ground. ( ) animals are from Africa? . . . ( ) are tigers from? . . . ( ) there three pandas in the zoo? , there are. aren’t. don’t know. ( ) many kinds of animals are mentioned in the passage? . . . 二.单项选择(每小题1分,共15分) ( ) 21. The little boy _______ play_______ piano. But he can sing a song . ’t, the , a ’t, / , the ( ) 22. ---Is this _______ bike? ---No. ________ is black. , My , Mine , Mine , My ( ) 23. ---Would you like _______ to my party? ---Great. I’d love to. come ( ) 24. Not only Jim but also his parents ____ going to visit the Forbidden City ______ the morning of Sunday. , in , on , on , in ( ) 25. Mrs. Li is looking forward to _______ her daughter. She studies in Australia now. see ( )26. There _______ a football match in the stadium this afternoon. have going to be going to have will have ( )27. By then ,machines will do _______ instead of(代替) people. heavy job heavy works heavy work heavy jobs ( ) 28. Every year there are _________ people come to visit our school. of of ( )29. ---I’m going to have a trip in England. ---__________. luck a good time ’m not sure a lot ( ) 30. They are going to go ________ their lessons. A. on B. over C. to D. by ( ) 31. She likes making friends _______ different people. A. for B. with C. to D. of ( )32. They are going to watch their country team _______ basketball tomorrow. A. play B. playing C. to play D. plays ( ) 33. Will there be any interesting news in today’s newspaper? –No, ______. A. there aren’t B. it won’t C. there won’t D. it isn’t ( ) 34. —_______ do you go to the music club? —Because I like singing and dancing. A. What B. How C. Why D. When ( )35. ---How much are the tomatoes? ---¬_______. yuan kilos yuan a kilo 三.完形填空。(每小题1分,共10分)[ Dear Dr Know, I'm not happy. I have too 36 rules in my family. I have to 37 at 6:00 every morning. I can't 38 my friends after school. 39 I have to 40 my dog for a walk. I can't watch TV on school nights. And I have to 41 in bed by ten o'clock. 42 the weekend, I have to clean my room and wash my clothes. Then I have to help my mother 43 dinner. Later I have to go to the Children's Palace 44 the piano. I never have any fun. 45 can I do? ( ) few ( ) to bed to school home up ( ) ( ) ( ) ( ) ( ) ( ). making ( ) learn ( ) 四.阅读理解。(每小题1分,共15分) ( A ) Found A set of keys. There is a toy bear on it, too. If you lost it, call me at 880-1167313. I’m from Middle School. Jack Lost An old bike. It’s black with a plastic(塑料的)basket. Its seat is yellow. If you find it, please return it to me. I live in the fifth floor in Building 2. Miller Lost I lost a novel(小说) written by Mr. Mo Yan. Its name is Frog. I want to find it. If you find it somewhere, can you give a phone to me? I’m from Class 6, Grade 7. Luis ( ) you lose a set of keys, you can call_______. Yan ( ) lose a(n) _______. of keys bike basket ( ) is from_______. . 2 Middle School fifth floor 2 6, Grade 7 ( ) is_______. kind of bike kind of key kind of basket novel ( ) one is true? lost a set of keys. is a toy monkey on that set of keys. found a novel Yan is a writer. (B) Richard is a super pop singer. He is very busy every week. This is his schedule (日程表) for next week. Monday Sing in New York Tuesday A concert in London Wednesday Meet Tina (his girl friend) in Sydney Thursday Go to his mother’s birthday party in Los Angeles Friday Sing for a group of sick people in a hospital in Paris Weekends Relax at his home ( ) is going to sing in ________ on Monday. ( )’s going to be in England on ________. ( )’s going to be with ________ on Wednesday. mother father daughter girl friend ( )’s going to be in Los Angeles for ________ on Thursday. concert sick people mother’s birthday girl friend’s birthday ( )’s schedule is very ________. ( C ) It is sunny and warm today. A dog is sleeping. A panda comes to the dog and says,"You are a lazy dog. You don't do anything every day." "What do I need to do?" asks the dog. My owner gives me food to eat. My owner takes me out for a walk. My owner loves me. Ha! I eat, I go out for a walk and I sleep. I don't have any other things to do." But don't you have any ambitions(抱负)?"asks the panda. " Ambitions? What do I need ambitions to do? I'm just a dog," says the dog. "What ambitions do you have?" “I often go to other countries and make friends with people there. So I can show the world that Chinese are friendly,” answers the panda. "Oh," says the dog,"maybe I should(应该)do something for others." 56. What is the dog doing in the sunny day? A. Walking B. Playing games C. Eating D. Sleeping 57.文中划线单词owner 的中文意思是“__________”. A. 主人 B. 老板 C. 长辈 D. 失主 58. The dog has only thing(s) to do every day. A. one B. two C. three D. four 59. The panda goes to other countries to show . A. China has a long history B. China has all kinds of animals C. China people love animals D. China people are friendly 60. The story tells us . A. We should relax a lot on sunny days B. Everyone should have ambitions C. Pandas are clever animals D. We should love animals. 五、补全对话(每小题1分,共5分) 从方框中选择最佳选项完成对话, 有两项多余。 A. I will live in space. B. What will you do there? C. Oh, I think life will be better. D. I will take a spaceship(宇宙飞船)to get there. E. Will you have a good time there? F. Maybe everyone will enjoy this life there. G. Will there be banks, schools and shops? A: What do you think your life will be like in ten years? B: 61 A: Where will you live? B: 62 A: How will you get there? B: 63 A: Will life be interesting there? B: Yes, I think it will be more interesting. A: 64 B: Of course there will be. I think the supermarket will have all kinds of things and we can take anything for free. A: 65 B: I agree(同意). People will have a happy life there. 61. ______62. ______63. ______64. ______65. ______ 六、完成句子。(每小题2分,共10分) 66、每个人都想要一个干净的教室,就想家一样。 Everybody would like a clean classroom, _______ _______ home. 67、别担心汉语。 Don’t ________ ________ Chinese. 68、玲玲打算在五一假期间玩的开心。 Lingling is going to ________ ________ during the My Day holiday. 69、天空中可能会有交通拥堵。 Maybe there will be ________ _________ in the air. 70、努力吧!你的梦想会实现的。 Work hard and your dream will _______ _______. 七、任务型阅读。(每小题1分,共5分) There is a man and he calls the police when he loses a pet. He always asks the police to help him look for it. “I lost my pet bird ,” he tells the police one day. “ she can sing songs. Can you find her for me?” “I am sorry,” the policeman says, “(A)It is not our job to look for birds. If (如果)you leave a window open in your house, the bird may come back .” Another time( 又一次),the man calls the police and says , “ I lost my cat.(B)你能帮我找到他吗? He can talk.” “I am sorry.” the policeman says, “It is not our job to look for cats. You should put the telephone down quickly. Maybe he is calling you now and tells you where he is.” 71. The underlined word(划线词)“it” refers to(指的是)_________ _________ in English. (一空一词) 72. 将文中(A )处划线句子译成汉语。 _________________________________________________________ 73.将文中(B )处划线句子译成英语。 _________________________________________________________ 判断下列句子正(T)误(F). ( )74. The man says his pet bird can sing . ( )75. The policeman agrees(同意) to help the man look for his cat. 八、综合填空。(每题1分,共10分) Johnson is an American. He has many(76)p_________for his holiday. He is going to China(77)b_________ plane for travelling. He is going(78)s________ in China. He is going to watch a table tennis match and he wants to(79)c________ the players. He is going to take a walk around a(80)c_______and(81)c_______litter.(82)B_____ he thinks it is meaningful. And he(83)h______ that the weather will be fine in Beijing every day. If it(84)r_____,it will be wet and he will not be(85)a_______ to do that. 九、书面表达。(共10分) 五一劳动节就快到了,下面是李明的一些计划和打算,请你根据所给内容,写一篇60字左右的短文,可适当发挥联想,增加内容。开头已经给出,不计入字数。 1. 五月一日早上做一些阅读,因为他喜欢读书;然后完成家庭作业。 2. 下午会帮父母做一些家务,例如打扫房子,做饭等。 3. 晚上会去参加英语俱乐部,因为他英语说得不太好。 4. 五月二日,将和他的父母一起去中洲公园(Zhongzhou Park)进行野餐,然后和父母一起在公园骑自行车。 May Day is coming. Li Ming will have a busy holiday. It’s going to be a busy holiday but great fun! 七年级英语第二学期月考试卷答案 一、听力答案:(每题1分,共20分) 1~5 CAACD 6~10 ABACA 11~15 AABBA 16~20 BACBB 二、单选:(每题1分,共15分) 21-25 ABBCD 26-30 BCDBB 31-35 BACCA 三、完型 (每题1分,共10分) 36-40 ADCBB 41-45 DCBBD 四、阅读 (46-55每题2分,共20分) 46-50 ACDDD 51-55 ABDCB 56-60 DACDB 五、补全对话(每题1分,共5分) 61-65 CADGF 六、完成句子。(每空1分,共5分) 66-70 just like worry about enjoy herself traffic jams come true 七、任务型阅读(每题1分,共5分) 71. his pet/the pet 72. 找鸟不是我们的工作。 73. Can you help me ( to ) find him? 74. T 75. F 八、综合填空。(每题1分,共10分) 76-85 Plans by sightseeing cheer country collect Because hopes rains able 九、书面表达。(10分)略猜你喜欢: 1. 七年级英语下册第二次月考考试题 2. 七年级下册英语第二次月考试卷 3. 2016七年级英语3月月考试卷 4. 七年级英语月考试题及答案 5. 七年级英语上册第二次月考试题及答案

初一英语期末考试卷

初一英语期末考试将至,背熟课本单词和熟读课本内容的同时,别忘记了做英语试卷题。以下是我为你整理的初一英语上册期末试卷,希望对大家有帮助!

I. 听力技能

(共2部分,计25分)

A) 对话理解。根据你听到的对话,从A、B、C三个选项中选择最佳选项回答问题。(共15小题,计15分)

听下面五段对话,每听完一段对话后,回答一个小题。

( )’s that in the boy’s book?

A. A letter. B. A CD. C. A photo.

( ) would Bob like to have?

A. Some rice. B. Some bread. C. Some chicken.

( )3. What time is it now?

. B. 7:40. C. 8:20.

( )4. What animal is Peter’s favorite?

A. The elephant. B. The panda. C. The dog.

( )5. What can you see in the box?

A. Orange. B. Apples. C. Bananas.

听第六段对话,回答6-7小题。

( )6. Whose shirt may it be?

A. Jack’s. B. Tom’s. C. Jean’s.

( )7. What’s the girl’s favorite color?

A. Green. B. Yellow. C. Pink

听第七段对话,回答8-9题。

( )8. What does Mr. Smith look like?

A. He is tall. B. He has big ears. C. He has small eyes

( )9. Where does Mr. Smith come from?

A. England. B. Canada. C. The USA.

听第八段对话,回答10-12小题。

( ) is the photo?

A. In Carl’s box. B. In Carl’s hand. C. In Carl’s schoolbag.

( )11. Carl thinks the photo is _____?

A. Mary’s B. Jack’s C. Jane’s

( )12. Jane is Carl’s ______?

A. sister B. friend C. classmate

听第九段对话,回答13-15小题。

( )13. What class is Susan in?

A. Class 2, Grade 7. B. Class 1, Grade 8. C. Class 2, Grade 8.

( )14. What does Susan look like?

A. She has a big nose. has long hair. C. She has blond hair.

( )15. What clothes is Susan in?

A. A pink T-shirt and green pants.

B. A blue T-shirt and blue pants.

C. A pink T-shirt and blue pants.

B)短文理解。听短文,根据所听到的内容,从A、B、 C三个选项中选择最佳选项回答问题。(共5小题,计10分)

( )16. What do Carl’s grandparents do?

A. Workers. B. Farmers. C. Cooks.

( )17. Carl’s father looks _____ ?

A. strong B. kind C. young

( )18. Where does Carl’s mother work?

A. In an office. B. In a school. C. In a hospital.

( )19. What’s Carl’s favorite food?

A. Fish. B. Chicken. C. Bread.

( )20. _____ people are in Carl’s family?

A. Three. B. Four. C. Five.

II. 阅读技能

(共5个部分,计50分)

完形填空。阅读下面的短文,掌握大意,从所给的A、B、C三个选项中选出一个最佳选项。(共10小题,计10分)

(A)

Bill is thirty years old. He works in a (21). He is a teacher. He goes to work (22) Monday to Friday every week. Usually he (23) at six in the morning. After (24), he takes a bus to work. He is a(n) (25) teacher, so he often plays basketball, ping-pong and volleyball with his students. He has (26) classes every day. Two are in the morning and two are in the afternoon. He likes his students and his students like (27), too. He often plays sports, so he is very (28). He usually eats dinner at home and then he watches TV for two (29). On weekends, he often goes to the movies. He (30) they are interesting.

21. C. schoo l

22.

23. up homework home

24.

25. .

26.

27.

28.

29.

30.

(B)

根据材料内容判断文后句子的正、误。正确的在括号里写“T”, 错误的写“F”。(共5小题,计10分)

Name Age Looks Clothes Favorite Food

Du Li 13 black hair

a small nose

big eyesx k b1 . co m a pink dress

white shoes fish

Sandy 12 a round face

big eyes

a small mouth a red skirt

a green shirt hamburgers

David 13 brown hair

a big nose

big eyes a white T-shirt

blue pants chicken

根据短文内容,判断正(T)误(F)。

( )31. Du Li, S andy and David are all thirteen years old.

( )32. Du Li, Sandy and David all have big eyes.

( )33. Du Li has black hair and she is in a red skirt.

( )34. Sandy is in a green shirt and she likes fish very much.

( )35. David has a big nose and chicken is his favorite food.

(C)

阅读下面的短文,并根据短文内容回答所提问题。(共5小题,计10分)

I have a good friend in my class. His name is Jim. He is from England. He is in China with his parents. He likes China. He likes Chinese food very much. He has breakfast at home. He eats an egg. bread and milk for breakfast. He doesn’t like fish. He has lunch at home. The lunch in his school is delicious. He can have different kinds of food for lunch. He eats rice, meat and vegetables. Sometimes he has noodles and dumplings. He has supper at home with his friends. Sometimes he goes out to eat with his friends. He has meat , vegetables and fruit for dinner .

36. Where is Jim from?

__________________________________________________

37. What does Jim like about China?

__________________________________________________

38. Does Jim have lunch at home?

__________________________________________________

39. How is the lunch in Jim’s school?

__________________________________________________

40. What does Jim have for dinner?

__________________________________________________

(D)

阅读下面的短文,从每小题所给的A、B、C三个选项中选出最佳选项。(共5小题,计10分)

It is Sunday today. Anna goes shopping with her mother. She wants her mother to buy a new coat for her. In Hongxing Shop, she finds a green coat. She tries it on. It’s too small. She wants a big one, but the big one is not green. Anna doesn’t like other colors.“Let’s go to another(另外的) shop to have a look.”her mother says. Then they go to Jiajiale Shop. The shop is big and they see many kinds of coats in different colors and sizes(尺码). Anna tries on a green one. It looks ni ce on her. So they take it for forty-five yuan.

( )41. Anna goes shopping with her ____.

A. mother B. father C. friends

( )42. Anna wants a new ____.

A. shirt B. skirt C. coat

( )43. The green coat is too ____ for Anna in Hongxing Shop.

A. long B. big C. small

( )44. ____ is Anna’s favorite color.

A. Blue B. Green C. Red

( )45. Anna’s new coat is ____ yuan.

A. 40 B. 45 C. 50

(E)

阅读下面的短文,从每小题所给的A、B、C三个选项中选出最佳选项。(共5小题,计10分)

Ted Best is an English boy. Now he is a student of Yuying Junior High School. He is in Class Six, Grade Seven. He has a dog. It’s black and white. The dog is very clever. Ted likes it very much. The dog’s favorite food is bone (骨头). Every day Ted goes home. He meets the dog on his way home.

Ted’s friend, Jenny is an American girl. She is in the same grade, but in different classes. She has a cat. It’s black and white, too. Fish is the cat’s favorite food. Jenny often buys some fish for her cat.

( )46. Where is Ted from?

A. England. B. American. C. Canada.

( )47. What’s Ted’s dog’s favorite food?

A. Meat. B. Bone. C. Rice.

( )48. Where does Ted meet his dog?

A. At Ted’s home. B. At school. C. On his way home.

( )49. Ted’s friend is ____.

A. Mr. Best B. Mr. White C. Jenny

( )50. What grade is Jenny in?

A. Gra de Six. B. Grade Seven. C. Grade Eight.

III. 写作技能

(共四个部分,计45分)

A)词汇运用。(5分)

一、单词拼写。根据英语首字母或所给汉语提示写出短文中所缺单词的正确形式。(共5小题,计5分)

(51)_________ (今天) is Sunday, but Grace is busy(忙碌的). She has to (52)_____ (帮助) her mom cook breakfast. And then she does her (53) _________ (家庭作业). After that, Grace goes to shop to (54) b____ some food and drinks. In the afternoon, Grace goes to (55) v_____ her grandparents with her pa rents. Her grandparents are very happy to see them. Grace and her cousin play games the whole(整个的) afternoon. Grace has a good time in her grandparents’ home.

二、综合填空。阅读下面的短文,在空白处填入一个适当的词或填入所给单词的正确形式。(共10小题,计10分)

Mr. and Mrs. Black have a happy (56)_________. They have a (57) _______ Jack and a daughter Mary. Mary (58)______ (be) a good girl. Every evening she usually has dinner with his parents. And Jack (59) _________ (like) sports. He often plays football and basketball (60)____ his friends. Sometimes (有时候) brother Jack goes to (61)______ (play) soccer and (62)_______ Mary also wants (63)__________ (go) with him. Mr. and Mrs. Black sometimes take (64)________ (they) daughter to watch his son’s soccer game, (65)________. Look, they watch the game under the tree.

B)句子翻译。阅读下面的短文,翻译画线部分的句子。(共5小题,计10分)

Tom is from the UK. 66.他是个英国男孩。He lives with his parents in Hong Kong. He is fifteen years old. He can speak Chinese. 67.他也会说英语。 He often helps his mother at home. 68. He can cook food well, and his classmates like the food. He likes Chinese food very much. likes to eat Chinese jiaozi a lot. 70. 他有很多好朋友。He often plays with them.

C)补全对话。根据下面的对话情景,在每个空白处填入一个适当的句子, 使对话的意义连贯、完整。(共5小题,计10分)

A: Hi, Susan. Can I ask you some questions(问题)?

B: Sure.

A: What’s your family name?

B: It’s Smith.

A: (71) ______________________________________________ ?

B: S-M-I-T-H.

A:(72) _______________________________________________ ?

B: It’s 320-7659.

A: What are your favorite animals?

B:(73) _______________________________________________ ?

A: I like lions, too. (74) _____________________________________________ ?

B: Yes, I’d like to. Let’s go to the zoo after school.

A: OK. See you.

B:(75) ____________________________ .

D)书面表达。(计10分)

请你根据下面思维导图提供的信息,以“My good friend”为题,用英语写一篇短文,介绍你的好朋友Linda.

要求:60词左右,短文须包含思维导图提示内容,可适当发挥。

参考词汇:交朋友make friends

一、听力技能.

1 2 3 4 5 6 7 8 9 10

C B A A C B A A B B

11 12 13 14 15 16 17 18 19 20

C B B C A B C C A C

二、 阅读技能

21 22 23 24 25 26 27 28 29 30

C A A A B C C B A A

31 32 33 34 35 41 42 43 44 45

F T F F T A C C B B

46 47 48 49 50

A B C C B

36.(He is from) England.

37.(He likes) Chinese food (very much).

, he does.

39. (It’s) delicious.

40. (He has) meat, vegetables and fruit (for dinner).

52. help 53. homework 54. buy 55. visit

57. son 59. likes 60. with

62. sister 63. to go 64. their 65. too

is an English boy.

can speak English, too.\ He can also spe ak English.

68.他能把食物烹饪得很好,他的同学们都喜欢这些食物。

69.他非常喜欢吃中国饺子。

70. He has many good friends.

71. How do you spell it?

72. What’s your telephone number?

73. (My favorite animals are) lions?

74. Would you like to go to the zoo with me?

75. See you

初一英语期末考试即将来临,为了帮助同学们备考英语期末考试,尝试着完成这份英语期末试卷吧。以下是我给你推荐的初一英语下册期末试卷及参考答案,希望对你有帮助! 初一英语下册期末试卷 一、单项选择(每空1分,共15 分) ( ) , we can sports in school an hour a day now. ---- Really? That sounds great. A. to play B. plays C. play D. playing ( ) goes to school after breakfast every morning. A. the, the B. \, \ C. the, \ D. \, the ( ) 3. We should do more in our lessons. A. exercise B. exercises C. exercising D. exercised ( ) 4. The fish . A. taste well B. tastes well C. taste good D. tastes good ( ) 5. --- How many teachers are there in your school? --- About four . A. hundred B. hundreds C. hundreds of D. hundred of ( ) 6. She often a train to school, sometimes she goes to school bus. A. takes, takes B. takes, by C. by, takes D. by, by ( ) 7. It will you a long time a language well. A. spend, learn B. to spend, to learn C. take, to learn D. take, learn ( ) 8. Li Tao is from a small village. A. a 11 years old boy B. a 11-year-old boy C. an 11-year-old boy D. an 11 years old ( ) teacher is very strict us our homework. A. in, in B. with, with C. in, with D. with, in ( ) 10. The movie is , and we are really very when we see it. A. relaxing, relaxing B. relaxed, relaxed C. relaxing, relaxed D. relaxed, relaxing ( )11. There are many animals in the zoo. But some animal are scary. A. kinds of, kind of B. kinds of, kinds of C. kind of, kind of D. kind of, kinds of ( )12. There are a lot of in our school. A. woman teachers B. women teachers C. woman teacher D. women teacher ( ) 13. –Is the woman your teacher? -- Yes, she teaches Chinese. A. us B. our C. ours D. we ( ) 14. – Excuse me. Is there a post office near here? -- . A. Yes, it is. B. No, this isn’t. C. Yes, there is. D. No, there aren’t. ( ) 15. – Have a nice weekend. -- . A. That’s OK. B. No problem. C. You’re welcome. D. Thank you. 二、完形填空(每空1分,共10 分) People in different 16 like to eat different foods. 17 kind of food people in the UK 18 to eat? Fish and fried 19 chips are very 20 in the UK. Sometimes people 21 them at home, but they usually go to a fast food 22 to buy them. They can have them 23 the restaurant or 24 them away home. Sometimes, they 25 like to eat them in a park. ( )16. A. country B. countries C. city D. town ( ) 17. A. Who B. When C. What D. How ( ) 18. A. would like B. like C. likes D. enjoys ( ) 19 A. potato B. potatoes C. potatos D. tomatoes ( ) 20. A. cheap B. large C. popular D. dear ( ) 21. A. cooking B. cook C. cooks D. to cook ( ) 22. A. restaurant B. library C. supermarket D. mall ( ) 23. A. in B. on C. under D. from ( ) 24. A. get B. bring C. take D. to take ( ) 25. A. and B. also C. too D. either 三、阅读理解(每空2 分,共30分) A We had a terrible school trip last week. Some students were late. Then we waited half an hour for the school bus, but it didn’t arrive. Finally, our teacher took us to take the subway. It took over an hour. When we arrived at the zoo, we were all tired and hungry. We wanted to see dolphins, but there weren’t any. There were some really smart seals(海豹), but we didn’t see the show because we arrived too late. We forgot our cameras, so we didn’t take any photos. Then it started to rain, and no one had an umbrella. We didn’t see many animals because of the rain. So we went for lunch. So we ate hamburgers. My friends also had some ice—cream. But I didn’t have any, because I don’t like it. I wanted French fries, but there weren’t any in the shop. They were sold out! Finally we went back to school. We were wet, tired and stressed out. I didn’t enjoy my school trip at all. ( ) did the teacher take the students to take the subway? A. Because the students didn’t want to have the trip. B. Because the school bus didn’t arrive. C. Because the students didn’t like taking the school bus. D. Because the students didn’t take cameras. ( ) 27. The students were when they arrived at the zoo. A. excited B. happy C. tired D. lucky ( )28. What animal show was there at the zoo? A. Dolphin show. B. Seal show. C. Monkey show. D. Panda show. ( )29. How was the weather after they arrived at the zoo? A. Sunny. B. Cloudy C. Windy D. Rainy ( )30. What did the writer think of the school trip? A. Terrible. B. Great. C. Not bad. D. Fun. B It’s easy to get lost(迷路) in strange city. Here are some tips(方法) to help you find the way and get to your destination(目的地). Take a map with you when you go to a new place. If you lose your way, look at the map, you’ll find the place where you want to go. If you still can’t find the place on the map, ask the police for help. They will tell you the right way to go. Remember the name of your hotel and the street where it is. If you can’t find the way back, take a taxi. I hope these tips can help you. ( ) 31. What does the underlined word “strange” mean? A. 熟悉的 B. 美丽的 C. 现代的 D. 陌生的 ( ) 32. How many tips are there in the passage? A. One. B. Two. C. Three. D. Four. ( ) 33. What should you take with you when you go to a strange city? A. A taxi. B. A map. C. A watch. D. A dictionary. ( ) 34. You should ask for help if you lose your way. A. a man B. a woman C. the police D. your teacher ( ) 35. Which is the best title(标题) of the passage? A. Get lost in a large city B. Some useful tips C. Never go to a strange city D. A map is useful C Hi, I’m a middle school student. I’m Judy. In my class there are thirty-eight students. How do they go to school every day? Seventeen of them take buses because they live far from the school. Six students walk. They think walking is good exercise. Fifteen students ride their bikes to school because they like riding bikes very much and their homes aren’t far from the school. No one goes to school by car. How do I go to school? Ha, I’m one of the fifteen. ( ) 36. There are students in Judy’s class. A. 29 B. 30 C. 38 D. 45 ( ) 37. Six students think is good exercise. A. running B. walking C. swimming D. riding ( ) 38. students ride their bikes to school. A. 17 B. 6 C. 10 D. 15 ( ) 39. Judy goes to school . A. on foot B. by bike C. by bus D. by car ( ) 40. No one gets to school . A. by car B. by bike C. on foot D. by bus 四、选用方框里的句子补全对话(每空1分,共5分) A: Hi, excuse me. ( ) B: Yes. 41 . ( ) A: Well, I’m new in town. 42 ( ) B: Yes, there is. It’s on Bridge Street. 43 ( ) A: Oh••• 44 B: It’s not too far from here. I can walk with you. A: Oh, that’s great! Thanks so much. ( ) B: 45 五、句型转换(每空2分,共10分) 46. You can’t wear sunglasses in the classroom.(改为祈使句) sunglasses in the classroom! 47. My weekend was boring.(对划线部分提问) your weekend? 48. Li Ming did his homework last night. (改为否定句) Li Ming his homework last night. 49. How’s the weather? (同义句转换) the weather ? 50. Joe sits on my left. Lucy sits on my right. (合并为一句) I sit Joe Lucy. 六、词语运用。(每空1分,共10分) (A)根据句意及首字母提示完成单词。 51. N and ten is one hundred. 52. Everyone must follow the school r . 53. If you want to borrow(借) a book, you can go to the l . 54. The house is too e . We can’t buy it. 55. Liu Huan is a famous m . I love his songs. (B)用所给词的适当形式填空。 56. I enjoy (skate)on the ice in winter. 57. Many children like to eat (candy). 58. At the zoo, Jane (take) many photos along the way. 59. Look! Mike (use) a computer . 60. It took me an hour (do) my homework. 七、 完成句子。(每空1分,共10分) 61. 我们必须准时到达学校。 We must get to school . 62. 我的父亲很喜欢写故事。 My father likes to very much. 63. 请把这些肉切碎。 Please the meat! 64. 如果你来,我会和你一起去。 If you , I go with you. 65. 你想要多大碗的? would you like? 八、书面表达。(10分) 根据下面的提示写一篇短文介绍你上周末的厦门之旅。 1. 上周末和父母一起去了厦门,住在叔叔家。 2. 星期六叔叔带我到厦门大学(university)参观。 3. 星期天参观了鼓浪屿(Gulangyu)和南普陀庙(South Putuo Temple). 4. 周末玩的很愉快。 初一英语下册期末试卷答案 一、单项选择:(15分) 评分标准:每小题1分,共15分。 1-5. CBBDA 二、完形填空 (10分) 评分标准:每小题1分,共10分 16-20. BCBAC 21-25. BAACB 三、阅读理解(30分) 评分标准:每小题2分,A、B、C三篇共30分 26-30. BCBDA 31-35. DCBCB 36-40 CBDBA 四、交际运用:(5分) 评分标准:要求写序号,写句子不给分,每小题1分,共5分。 41-45 DCEBA 五、句型转换:( 10分) 评分标准:按空给分,每空1分,每小题2分,共10分。 46、Don’t wear 47、How was 48、didn’t do 49、What’s like 50、between and 六、词语运用: (10分) (A)根据句意及首字母提示完成单词。 评分标准:每小题1分,共5分 51、Ninety 52、rules 53、library 54、expensive 55、musician (B)用所给词的适当形式填空。 评分标准:每小题1分,共5分 56、skating 57、candies 58、took 59、is using 60、to do 七、 完成句子(10分) 评分标准:按空给分,每空1分,每小题2分,共10分。 61、on time 62、write stories 63. cut up 64、come, will 65、What size 八、 书面表达:(10分) 评分标准: 1、作文包含所要求的内容,并有适当的发挥,句子通顺,条理清楚, 没有语法和单词错误可以给予10分。 2、作文包含所要求的内容,句子通顺,存在单词和语法错误,可以给7--9分。 3、内容不完整,句子较通顺,老师酌情给分。猜你喜欢: 1. 七年级英语期末备考卷及答案 2. 新人教版七年级英语下册期末测试卷 3. 七年级英语期末考试卷附答案 4. 2017七年级英语下册期末试卷及答案 5. 七年级下册英语期末考试卷及答案

七年级英语 期末考就要到了,我亲爱的朋友,祝你考试顺顺利利,考出理想成绩!下面是我为大家精心整理的七年级英语下册期末测试题,仅供参考。七年级英语下册期末测试题 一、选择题(15*1’=15’) me. Is________camera yours? ---Oh, yes. I am looking for it everywhere. Thank you very much. A. a D./ was your trip to Lianyungang? ---Wonderful. On the way_______Huaguo Mountain, we saw some houses different from ours. A. to play the piano when you were five years old? ---No. But now I am good at it. is said that a lot of cities took action to stop people from Chinese style road crossing(中国式过马路). ---___________wonderful the news is! an a there______interesting in today’s newspaper? ---No. But there is some important news. happened to you yesterday? ---A football______me when I was on the street. will the weather be like tomorrow? ---It will be rainy, so you should keep the windows___________. you _____________a tent by yourself? ---Sorry. It’s a little difficult for me. on up down off 9. ---Where is the sick boy now? ---He is ill_____________. hospital the hospital hospital the hospital was hurt in the fire? ---__________man called John. 79-years-old 79-year-old 79 years old 79 year old is Nancy’s hobby? ---She likes singing. I often hear her_________in the next room. sing coat looks nice. How much did you____________it? ---Two hundred yuan. in to for , __________should I take the medicine? ---Twice a day. time often long soon do you want to help? ---Children in poor areas, because many of them are_______poor_____go to school. ; to ; to ; too ; can’t am poor at English. I am unhappy all day. ---___________. If you work hard, you will learn it well. out out on on 二、完形填空(15*1’=15’) Dad lost his job . When Christmas came that year, we had no money 16______. On Christmas Eve, Dad took me out into the park. I had a blanket, so I 17_______ Dad to share with me, but he said, “No, thanks. ”We 18______ at the sky. Those beautiful stars were one of the 19______presents for people like us who lived out in the wildness. Rich city people, he’d say, lived in 20______ flats, but their air was dirty, so they 21_____ not even see the stars. “ Pick up your favorite star.” Dad said that night. He 22_______ me I could have it for keeping. He said it was my Christmas 23______ . “You can’t give me a star,” I said, “ 24_______ has the stars.” “That’s right,” Dad said, “But you can suppose(假设) it is yours. And when you do that, it is yours.” I looked at the stars, and tried to find out 25_______ was the best one. Then, I found one. “I want that one.” I said. Dad smiled. “That’s Venus(金星),” he said, “Venus was only a planet. She looked bigger and brighter because she was 26_______ than the stars. Poor old Venus didn’t even make her 27_______ light. She shone only from reflected(反射)light.” “I like it anyway,” I said, “I loved Venus even 28_____ that Christmas. You could see it in the early evening, and if you got up early, you could still see it in the morning, after all the stars had disappeared(消失).” Dad said, “Ok, it’s Christmas. I accept(接受) your decision. You can have it if you want.” And he gave 29______ Venus. We all laughed about the kids who think the Santa myth(圣诞 传说 )is true and only got a lot of cheap toys. “Many years later, when all the things they got are broken and long forgotten,” Dad said, “You will have your star 30_____the end. ” 16. A. at first B. at once C. at all D. at least 17. A. asked C. reply D. replied 18. A. looked up B. looked for C. look out D. look after 19. A. strange B. special C. bad D. famous 20. B. small C. old D. poor 21. A. needed B. could C. should D. must 22. A. told B. tells D. spoke 23. A. dinner B. card C. party D. present 24. A. Everybody B. Somebody C. Anybody D. Nobody 25. A. which B. why C. when D. how 26. A. slower B. farther C. closer D. higher 27. A. small B. bad C. strongly D. own 28. A. when B. before C. after D. until 29. A. me B. us C. we 30. A. before B. never C. already D. till 三、阅读理解(15*2’ =30’ ) (A) It’s Kids’ Week at Big Mountain Ski Resort! From December 27 to January 2 Children can ski for half price! Kids will also receive: —Free hat! —Coupon(礼券) for one free hot chocolate at the Pine Lodge Café! —Free 1-hour ski or snowboard lesson! Must be 14 or younger. Don’ t forget to join the snowboard contest. Just put your name and address on a piece of paper and put it in the game box in the Pine Lodge Café. If we pick your name, you will win a new Shark snowboard! Come to Big Mountain for lots of fun and great prizes! 31. This advertisement is for _____. A. school students B. the students at 14 C. the kids who are 14 or younger D. all the kids 32. A kid will get _____ at Big Mountain Ski Resort. A. free hat B. coupon for one free hot chocolate C. free 1-hour snowboard lesson D. all of A, B and C 33. If a 13-year-old boy usually pay £60 to ski, he will pay ___ on December 28th. A. £120 B. £60 C. £30 D. £15 (B) It was very cold came a black car and it stopped in front of a shopping young woman was in the was hungry and wanted to buy a hamburger to eat, but she didn't want to go out on a so(如此)cold that time she found a boy playing in the street. "Come on, boy!" she said to the boy."Do you know how much a hamburger is in the shopping mall?" "Two dollars each," the boy answered. She gave the boy four dollars and asked him to buy two hamburgers."One is for you and the other is for me," she said. Some minutes(分钟)later, the boy came was eating a gave the young woman two dollars and said," Sorry, there's only one hamburger left(剩下的)." black car stopped because_______. weather was very cold. was broken(坏的). woman found a boy playing in the street. woman wanted to buy a hamburger hamburger in the shopping mall was_______. A.$2 B.$4 C.$1 don't know. young woman had _______. hamburger hamburger hamburgers hamburgers the article, we can know the story happened(发生)_______. winter summer a cool day D. on a hot day (C) Qi Haoran, a Junior 1 student, was quite busy over the past winter holiday and not just with homework. Qi, together with 10 other classmates made a volunteer group to call on people to join the Clean Your Plate Campaign (“光盘行动”). These students went to many restaurants and told people that saving food is important. “Excuse me, do you know that 950 million people around the world still don’t have enough food to eat? Please don’t waste(浪费) food.” They would say this kind of thing hundreds of times every day. The Clean Your Plate Campaign began on the Internet in January. It calls on people to reduce(减少) food in recent years had experienced(经历)serious problems with wasted food. CCTV reported in January that the food Chinese people waste every year is enough to feed 200 million people for a year. Chinese people are well-known for being hospitable (好客的). Many even feel that they lose face if their guests have eaten all the food. Luckily, the campaign has got a lot of support. In a restaurant in Xinjiang, the owner give the guests who have eaten up all that they ordered a sticker (贴花). People can enjoy a free meal when they have 10 stickers. More than 750 restaurants in Beijing have begun to offer smaller dishes and encourage their guests to take leftovers (剩饭剩菜) home. To reduce food waste is a big task, and it needs time. It’s important that everyone does a little, just like Qi. Did you finish your meals today? from Qi’s High School volunteered to call on people to join the Clean Your Plate Campaign. Clean Your Plate Campaign started_____________in January. a big restaurant a CCTV programme a high school the Internet to the CCTV report in January, the food we waste every year can___________. us enjoy free meals people from going to the restaurants 200 million people for a year us become generous restaurant owner in Xinjiang__________to support the campaign. the guests a sticker if they ate up the food the guests 10 stickers if they ordered a lot smaller dishes to the guests customers to take leftovers home (D) Many of us don’t know that eye care(眼护理) is important. It is said that if you take care of your body, then you can surely be healthy. That is why our eyes should be given a lot of care. Natural eye care should be put in a number one place. There are several causes(原因) leading to poor eyesight(视力) like not enough food, genes(基因) and aging(老化). Televisions, computers and reading are also the causes of having poor eyesight. If you happen to work in front of the computer, it is best to take a break every once in a while. Something dirty can cause redness and they will make you uncomfortable. It is bad for your eyes, too. If this happens, the best way is to clean your eyes by using cold water. You must also try your best to protect your eyes from harmful(有害的) things. For example, sunglasses are not just for fashion but they can also serve as a great way to protect your eye-sight from UV rays. Eating healthy foods will do good to your eyesight. Remember that vitamin A, C and E are good for eyes. Try to eat food groups that have these vitamins. And you should do eye exercises because exercise protects your eyesight, too. If a person exercises regularly(有规律的) and eats the right kind of food, his eyes will stay in good condition(状况) for a long time. All above are natural ways of eye care that help us keep healthy eyes. Being happy all the time can be helpful to a person’s eyesight, too. In a word, eye care is very important, no matter how old a person is. 42. __________ is the most important way to protect our eyes. A. Natural eye care B. Taking medicine C. Seeing a doctor D. Being happy all the time 43. What should you do if you have to work in front of the computer? A. Eat healthy foods. B. Clean the eyes by using cold water. C. Wear a pair of sunglasses. D. Have a rest after working for a while. 44. What do the underlined words UV rays mean? A. 沙尘 B. 闪电 C. 紫外线 D. 超声波 45. Which is the best title of the passage? A. Ways of eye care B. Ways of eye exercises C. Ways of Being Happy D. Ways of Being Healthy 非选择题 四、单词拼写(10*1’=10’) A.根据句意和汉语提示,在空白处填入适当的单词 box is____________________(锁着的), so we don’t know what’s in it. police__________________(搜索)the man and found a gun inside his pocket. all know that one hundred year is a___________________(世纪). daughter is very clever but sometimes she is_______________(粗心的). young man likes____________________(打架), so nobody likes to make friends with him. B.根据首字母及上下文提示补全单词,完成短文。 Chocolate lovers know that eating chocolate makes them happy. Now they may also b__________(51) smarter by eating chocolate. Scientists let a group of people eat 85 grams of milk chocolate and asked another group to eat no chocolate at all. After 15 minutes, the group that ate chocolate were a__________(52) to memorize(记住) more words than the group that did not eat chocolate. They b__________(53)that something in chocolate could make people smarter. Of course eating chocolate doesn’t guarantee(保证) you will p_________(54)every exam(考试). You shouldn’t eat too much chocolate because there is too much f___________(55)in it. If you want to do well in your study, you will surely need to work hard. 五、用方框中所给词或词组的正确形式填空(10*2’=20’) lose one’s way; take part in; not...any more; get away; grow; take...out of...; be afraid of; hear from; look out; at least book ______________his schoolbag and put it on the table. , the police caught the thief when he decided_______________________. eyes are the same size from birth, but our nose and ears never stop____________________. ! Don’t drive too fast. old man______________________on the street, so he asked me for help. parents once a month. can______________________do anything bad to the ’s our home. sports activities to keep fit next year. when you were young? should drink_____________________8 glasses of water every day. 六、完成 句子 (5*1=5’) (转身)but saw nothing. (尽力地通过)the door, but failed. should______________________(小心)fire. It can be very dangerous. are________________________(容易照料). you have a picnic, ____________________________(记得带上你的手机). 七、任务型阅读(5*1=5’) A plant has four main parts: flowers, leaves , stems(茎) grow above ground, and roots grow below ground. Roots bring water from the ground to the plant. The stem carries water from the roots to the leaves and flowers. It holds the leaves and flowers in the sunlight, too. Leaves make food for the plant. Why do plants have flowers? Plants have flowers because the flowers make seeds. New plants grow from seeds. People eat all parts of plants. For example, we eat the leaves of cabbage, the root of a carrot. Which part of rice do we eat? We eat the seeds. The seeds of the rice plant feed billions of people. What’s a billion? It’s 1,000,000,000. 阅读短文,完成下面的表格。 Parts of plants What is it for? An example to eat Bring water from the ground to the plant. Carrot Carry water from roots and hold the leaves and flowers. Celery(芹菜) Leaves Make for the plant. Flowers seeds. Broccoli 八、书面表达(20’) 根据提示内容写一封推荐信给校长。内容如下: 很聪明,学东西很快。 2.他很有思想,工作时考虑很周到。 3.他很乐于助人,经常帮老师拿东西,解答同学们的问题。 4.他是我们班里的优等生。 要求:50词左右(开头和结尾已给出,不计入总词数)。 Dear Headmaster, I would like to recommend Simon for this year’s Best Student Award. We all think that Simon should get the award. We look forward to hearing from you soon. Yours faithfully, Millie 七年级英语下册期末测试题参考答案 一、1-5 CABAB 6-10 ABBAB 11-15 ACBBC 二、16-20 CAABA 21-25 BADDA 26-30 CDBAD 三、31-33 CDC 34-37 DAAA 38-41 CDCA 42-45ADCA 四、A. B. 54. pass 五、took out of; to get away; growing; Look out; lost his way; hears from; will take part in; Were afraid of; at least 六、turned around; tried her best to go through; be careful with; easy to look after/take care of; remember to take your mobile phone 七、Roots; Stems; food; Cabbage; Make 八、 Dear Headmaster, I would like to recommend Simon for this year’s Best Student Award. Simon is very clever. He learns things very quickly. He is a thoughtful boy. He is very careful when he works. He is also helpful. He often helps teachers carry things and helps us with our problems. He is a top student in our class. We all think that Simon should get the award. We look forward to hearing from you soon. Yours faithfully, Millie 七年级英语期末测试题及答案相关 文章 : 1. 七年级英语期末试卷及答案 2. 七年级上英语期末试卷及答案 3. 七年级上册英语期末考试卷及答案 4. 人教版七年级英语上册期末试卷及答案 5. 人教版七年级英语上册期末试卷及参考答案

初一英语期中考试卷

七年级英语期中考试已进入复习阶段需要多做英语期中试卷来巩固知识,为了能帮助参加英语期中考试的同学们备考英语。以下是我给你推荐的七年级英语下期中试卷及参考答案,希望对你有帮助! 七年级英语下期中试卷 二、听较长对话,选择正确答案。(共6小题,计12分) 听下面两段长对话,每段对话后有几个小题,从题中所给的A、B、C三个选项中选出最佳选项。每段对话读两遍。 听第一段对话,回答第6至第8三个小题。现在,你有15秒钟的时间阅读这三个小题。 6. What does Mike have to practice this Saturday? A. The piano. B. The guitar. C. The drums. 7. Where is Rose now? A. In the sports club. B. In the music club. C. In her classroom. 8. What’s Rose’s telephone number? A. It’s 256-9811. B. It’s 256-8911. C. It’s 265-9811 听第二段对话, 回答第9至11小题。现在,你有15秒钟的时间阅读这三个小题。 9. What does the boy have to wear in the school? A. Sports shoes. B. A shirt. C. The school uniform. 10. What CAN’T the boy do? A. Eat in the dinning room B. Eat outside C. Eat in the classroom 11. Who are they? A. Classmates B. Mother and son C. Teacher and student 三、听独白,根据所听内容选择最佳答案。现在,你有20秒钟的时间阅读这四个小题。(共4小题,计8分) 12. Where does the monkey come from? A. China B. Japan C. Australia does the monkey like doing? A. sleeping B. jumping and running C. walking 14. What color is the tiger’s body? A. Yellow and light orange B. Orange with black rings C. Yellow with black rings 15. How long can the giraffe’s neck be? A. 1 meter B. 2 meters C. 3 meters 第二部分 笔试部分 四、阅读理解。(共两节,计40分) 第一节,(共15小题,计30分) 阅读下面材料,从题中所给的A、B、C、D四个选项中选出最佳选项。 A A woman was at the cinema, and she liked the film very much. But there was a man in the next seat, and he began looking on the floor under him. The woman was angry, but she can’t speak loudly then. So she whispered, “What are you doing there?” “What are you looking for ?” “A piece of hard chocolate,” the man whispered to her, “I dropped (掉落) it on the floor.” “A piece of chocolate?” the woman said angrily, “It’s dirty now! Take this and be quiet, please! I’m listening to the film!” She gave the man a big piece of chocolate. “But,” he man said, “my teeth are in the piece on the floor!” 16. The woman was _________. A. watching a TV play B. listening to the music C. watching a football match D. seeing a film 17. The man sat _________ the woman. A. behind B. in front of C. next to D. far from 18. The woman was angry because _________. A. the man spoke loudly B. the man didn’t talk to her C. she dropped a piece of chocolate D. she couldn’t listen to the film clearly(清楚地) 19. The word “whisper” means “speak in a _________ voice”. A. low B. high C. happy D. big 20. In fact the man was busy looking for his _________. A. glasses B. teeth C. shoes D. feet B Rules are everywhere. When you’re reading at the library, playing basketball on the playground or driving on the road, you will have to follow some rules. But do you know about any rules on a plane? If you’re on a plane in America, you may find the following rules. Exit Seat (紧急通道位) Criteria If you are sitting in an exit seat, read the information(信息) below. You must: ◆Be older than 15 years old and healthy enough to give help. ◆Be able to speak, read and understand English well. ◆Be able to find a safe way from the emergency exit (紧急出口). ◆Be able to help others away from the plane. You must not: ◆Travel (旅行) with a pet, such as a dog. ◆Travel with someone that needs special care, such as a child. If you cannot meet the criteria (标准)ask crew members (工作人员) to move you to another seat. You do not have to say why, just say, “I do not want to sit in an exit seat.” 21. The passenger sitting in an exit seat ________. A. must be under 15 B. needn’t speak English C. must be healthy enough D. needn’t help others 22. If you are sitting in an exit seat, you can NOT ________. A. travel with a friend B. bring a pet with you C. read Exit Seat Criteria D. move to another seat 23. This piece of Exit Seat Criteria may appear _______. A. on the playground B. on a train C. on the road D. on the plane 24. The Chinese meaning of the underlined word “meet” in this passage is probably “ ____”. A. 遇见 B. 符合 C. 相信 D. 开会 25. What’s the best title (题目) for the passage? A. Rules on a plane B. A trip to America C. How to help others on a plane D. Special care on a plane C One day, an American man Simon went to London to visit his friend, Rick. Rick told him that his flat(公寓) was on the first floor. When he arrived, Simon went straight to the first floor of the building. But a man told him that there was no Rick on that floor. Do you know why? In fact, the British (英国人) call the first floor of a building the ground floor. The floor above the ground floor is the first floor, but Americans would call it the second floor. The story shows that there are a few culture differences between Britain and America, though the British and Americans both speak English. The British don’t like showing their feelings. They hardly start a small talk with strangers. For example, on the train the British often spend their time reading newspapers or books. But Americans are quite different. They’re more active and easier to talk with. The British and Americans may use different terms for the same things. The British usually use “football” “holiday” and “lift”, but Americans like to use “soccer” “vacation” and “elevator”. 26. Simon went to London to ____________. A. visit his friend B. spend his holiday C. study English D. have a meeting 27. Which picture shows us where Rick’s flat was? A. B. C. D. Rick’s flat Rick’s flat Rick’s flat Rick’s flat 28. According to the passage, the British usually spend their time ______________ on the train. A. playing cards B. talking loudly C. doing some reading D. singing and dancing 29. What does the underlined word “terms” mean in the passage? A. 成果 B. 学期 C. 会议 D. 词语 30. What’s the best title for the passage? A. Culture differences between Britain and America B. A funny story about Simon C. Small talk in Britain and America D. British people and American people 第二节,任务型阅读(共5小题,计10分) 阅读以下五个名人的介绍,从A-F中找到与1-5题对应的内容,把名人介绍补充完整。 31. Liu Xiang was born in Shanghai in July, 1983. He is a great sportsman. 32. Ma Ji was born in Beijing in 1934. He was a famous crosstalk(相声) master. 33. Yang Liwei was born in Liaoning Province. He has a happy family. He loves his wife and his son. 34. Wang Yuan was born in 2000, Chongqing. He was a very famous teenage singer. 35. Lu Xun was a great writer. He had deep eyes, thick moustache and short hair. A. He used to be an excellent pilot with an experience of 1,350 hours’ flight. He is the first astronaut who flew in space in China. He has a dream that is to step on the moon. We’re proud of him! B. He did well in satirizing(讽刺)social problems and praising heroes. He not only inherited and develop the crosstalk, but also created his own style. He contributed a lot to China’s crosstalk art. C. He used to dress in a long suit. He smoked a lot while he was in deep thought. He hated the enemy, but he loved the people very deeply. He wrote a lot of novels and articles to fight against the enemy bravely. D. As a boy, he entered the children’s sports school and received a lot of strict training under his coach Sun Haiping. He won the gold medal in the 100-meter hurdles in the 28th Athens Olympic Games. E. He devoted all his life to his research work. He was awarded as the yearly top scientist in 2001. F. As a member of the poplar Chinese boy band TFBoys, he is a very popular singer now. At the same time, he was the first Chinese teenage star to go to the UN youth forum and gave a speech. 五、完形填空。(共15小题,计15分) 阅读下面的短文,掌握其大意,然后在每小题所给的A. B. 四个选项中选出以填入空白处的最佳选项。 Little May is a lovely and beautiful girl, but she had one 36 habit. If you asked her to do anything, she would say, “In a(n) 37 .” If her mother said, “ May, dear, 38 me my coat,” she would say, “ Yes, Mother, in a minute.” And even if her father called 39 for a ride(兜风), it was not 40 . She 41 did it at once what she was told to do. One day May’s bird was flying about the room. Someone went out and left the 42 open. May’s mother said, “ close the door, my dear.” “ Yes, Mother, in a minute,” said May. “I only want to 43 this story.” 44 the cat did not wait. She came in, and with one jump had the bird in her 45 . The book went on the floor, and May ran 46 the cat. Soon poor May came back crying with the dead bird in her hand. Her mother was 47 sad. But she said, “ My dear little girl, you 48 that many things may happen in a minute.” It was a sad 49 for May, but it was one she would not 50__. 36. A. relaxing B. boring C. funny D. bad 37. A. minute B. hour C. day D. year 38. A. help B. bring C. buy D. wash 39. A. him B. her C. it D. them 40. A. important B. interesting C. helpful D. different 41. A. never B. sometimes C. often D. always 42. A. house B. room C. window D. door 43. A. write B. finish C. know D. learn 44. A. And B. But C. Because D. So 45. A. hand B. foot C. arm D. mouth 46. A. after B. like C. with D. away 47. A. still B. also C. just D. even 48. A. say B. tell C. find D. see 49. A. class B. lesson C. time D. taste 50. A. remember B. forget C. lose D. keep 六、语法填空。(共10小题,计10分) 阅读下面材料,在空白处填入适当的内容或括号内单词的正确形式。 Many people like pandas very much because they are very cute. Most of them come ___51__ Sichuan, China. They have white hair, but 52 (they) ears, eyes, legs and shoulders are all black. Pandas are usually heavy, but they can climb trees like koalas. Their favorite food __53__ (be) bamboo. They like to eat a lot. It takes pandas a lot of time 54 ___ (eat) food every day. When pandas are full, they like ____55___(sleep). They sleep many 56 ___ (hour) every day. ____ 57 there are not many pandas in the world today. They are 58 great danger. Because people cut 59 __ too many bamboo trees, pandas can’t find much food. I think we must help them. We can plant more bamboo trees and be ___60____ (friend) to pandas. 七、单词拼写。(共10小题,计10分) 根据下列句子及所给首字母,写出各单词的完全形式(每空填一词)。 61. They usually eat dinner at a q to seven in the evening. 62. We all know that it is dangerous to c the road when the traffic light is red. 63. Your home is far away. Do you go to school by bus or by s_______? 64. My teacher is kind but she is s with us in our study. 65. Thirty and sixty is n . 66. Bill b his teeth every day at night before he goes to bed. 67. Jacky is a good student. He gets up early every day and never arrives l_____ for school. 68. It’s Dragon Boat Festival today. Zhu Hui really m_____ his family and wishes to have his mom’s delicious zongzi. 69. I work hard to make my dream come t . 70. I e watch TV or play computer games on Saturday evening. 八、书面表达(共1题,计10分) 假如你是Mike,你是一名中学生,根据下面表格中的内容,用英语写一篇短文,介绍你和家人上学及上班的情况,并谈谈你对家人乘坐不同交通工具的看法。 要求: 1.所写内容必须包括表格内的信息,可适当发挥。 2.词数:70词左右。 Who Mike Father Mother Where School TV station Radio station How By bus By car By bus How long 15 minutes 30 minutes 20 minutes 七年级英语下期中试卷答案 1-5 BBCBA 6-10 BAACC 11-15 CABAB 16-20 DCDAB 21-25 CBDBA 26-30 ABCDA 31-35 DBAFC 36-40 DABBD 41-45 ADBBD 46-50 ABDBB 51 from 52 their 53 is 54 to eat 55 sleeping/to sleep 56 hours 57 But 58 in 59 down 60 friendly 61 quarter 62 cross 63 subway 64 strict 65 ninety 66 brushes 67 late 68 misses 69 true 70 either Hi, I'm Mike. I'm a student in a middle school. Every day, my family members rise early and go to work at once. I usually ride to school in 15 minutes. It's a relaxing way for me. My father works at a TV station and it's a little far from my home. So he goes to work in his car every day. It takes him half an hour. My mother works in a radio station which is near a bus stop. Of course, my mother takes a bus to work and it takes her about 20 minutes. My mother and I always go to work(school) in a green way but my father doesn't. Driving a car is fast, but it can also make air pollution. So I'll ask my father to take a bus or a subway to work. Let's go to school in a green way!猜你喜欢: 1. 七年级英语下册完形填空测试题及答案 2. 人教版七年级下英语期末试卷及答案 3. 七年级英语第二学期期中考试题及答案 4. 七年级英语上册期中试题及答案 5. 七年级英语下册月考试卷及答案

七年级英语期中考试临近,历年的英语期中试题也可以拿来当复习资料,七年级英语期中考试要加油!以下是我给你推荐的七年级英语期中试题及答案,希望对你有帮助! 七年级英语期中检测试题 第I卷选择题 共80分 I. 听力测试25分 听句子,选择最佳答案每小题1分,共5分 B. Green C. Mary Green ,I am. B. Yes,I do. ’s nice. B. Thank you. C. Yes, it’s my watch. are my balls. B. Let’s play tennis. C. In my desk. sounds good. like apples. C. Yes , I do. 听一面对段对话和问题,选择最佳选择每小题1分,共5分 11. A. Yellow and red. B. Red and green. C .Yellow and green. 12. A. It’s under the bed. B. It’s under the desk. C. It’s on the bed. 13. A. No, she only has a bookcase. B. No, she doesn’t. C. Yes , she does. 14. A. We don’t know. B. No, he doesn’t. C. Yes, he does. and broccoli. B. Broccoli and potatoes. C. Potatoes and carrots. 听长对话,选择最佳选项每小题1分,共5分 Peter have puter games? A. Yes , he does. B. No , he doesn’t C. We don’t know. Grace play tennis? A. No, she can’t. B. Yes , she can. C. Yes , but she doesn’t like it. ’s the soccer ball? A. It’s on the bed. B. It’s under the chair. C. It’s under the bed. sport can’t Grace play? A. Soccer. B. Basketball. C. Volleyball. sport does the boy like? A. Soccer and tennis. B. Basketball and soccer. C. Tennis and volleyball. 听短文,根据所听内容选择正确答案每小题1分,共5分 ’s the girl’s name? A. Amy Brown. B. Amy Black. C. Susan Brown. food does the father like? A. French fries. B. Hamburgers. C. Vegetables. the father play soccer? A. No ,he doesn’t like it. B. Yes , he does. C. No, he only watches it on TV. does the girl play tennis with? A. Her father. B. Her mother. C. Her friend. does the mother have? A. Ten soccer balls. B. Ten tennis rackets. C. Three tennis rackets. II. 选择填空 15分 从每题A、B、C、D四个选项中,选出一个最佳答案。 26. -Hi, Alice ! What’s this in English? - It’s ________ Eraser. B. an C. the D. / 27. - Is that your ________? - Yes, she is. A. father B. uncle C. brother D. sister 28. This is my son. ________ name is Yang Yang . A. He B. She C. His D. Her 29. -What’s “联合国”in English? -It’s________. A. CCTV B. UFO C. BBC D. UN 30. My sister likes vegetables ________ dinner. A. on B. in C. for D. at 31. Please take ________ things to your sister, Tom. A. it B. this C. that D. these 32. Frank and Mary are my mother’s parents. They are my________. A. grandparents B. grandfather C. grandmother D. uncles 33. -________ your friend like English? -Yes, he does. A .Does B. Do C. Is D. Are 34. -Let’s watch TV. -No, that sounds ________. A. good B. boring C. fun D. relaxing father ________ a clock but he doesn’t like it. A. have B. has C. is D. does 36. -________ is my notebook, Mom? - It’s on the sofa. A. What B. How C. What color D. Where 37. -Hi, Mary! How are you? - ________ A. Yes, please. B. Thank you. C. I’m fine, thanks. D. How are you? 38. + = ________ apples. A. Four B. Five C. Eight D. Nine 39. The boys play ________ every day. A. soccer B. a soccer C. soccer ball D. the soccer ball 40. Please________ my watch to me, Sally. I’m in my room. A. take B. bring C. run D. need 41. Is this your ID card? Please ________ Nick at 489-6756. A. look B. watch C. sound D. call 42. -________ Jenny. Is this your ruler? - Yes, thank you. A. How are you? B. Thank you, C. Excuse me, D. Nice to meet you! 43. -Does your brother like ________? -Yes, he does. A. apple B. banana C. carrot D. salad 44. -Does Linda have a baseball? she has a basketball. A. Yes, she is B. No, she isn’t C. Yes, she does D. No, she doesn’t 45. - ________, Jim? - No, it isn’t. It’s her backpack. A. Is that your brother B. Is this your backpack C. Are these your backpacks D. Are those your brothers III.完形填空 10分 阅读下列短文,从文后各题所给的四个选项中选出一个最佳答案。 Hello, everyone! Do you know 46 ? My name 47 Cindy Brown. Cindy is my 48 name and Brown is my family name. I am a 49 .I am nine years old. My 50 is 667-1298. There are有 51 people in my family-my parents, my brother and I. We all like fruit and vegetables. They are 52 food. We often 53 them for lunch and dinner. I like books, 54 I don't like ball games. My brother, Peter Brown ,likes ball games but he 55 books. We both两个都love our parents and our family. 46. A. me C. my D. us 47. A. am B. are C. is D. be 48. A. last B. family C. one D. first 49. A. boy B. girl C. daughter D. cousin 50. A. bookcase B. watch C. answer D. telephone number 51. A. three B. four C. five D. six 52. A. healthy B. boring C. relaxing D. fun 53. A. play B. have C. watch D. call 54. A. and B. or C. but D. so 55. A. likes B. don't like C. like D. doesn't like IV. 阅读理解 每小题分,共计30分 阅读下面两篇短文,从每题A、B、C、D 四个选项中,选出一个能回答所提问题或完成所给句子的最佳答案。 A Li Ming: Excuse me, Wu Dong. Is that your dog? Wu Dong: Let me have a look. Oh, no, it is yellow. I think it’s Wang Hong’s. My dog is black. Li Ming: Wang Hong. Look at the dog under the tree. Is it your dog? Wang Hong: No, it isn’t. I don’t have a dog. I think it’s Ma Jun’ s . Li Ming: Ma Jun? Wang Hong: Yes, he’s my friend. Look! He’s there. Let’s go and ask him. Li Ming: OK! Let’s go. Wang Hong: Hi, Ma Jun! Is that your dog? Ma Jun: Oh, yes, it is. Wu Dong: It’s a nice dog! Ma Jun: Thank you. 56. The dog is ________ . A. Wu Dong’s B. Li Ming’s C. Wang Hong’s D. Ma Jun’ s 57. Wang Hong has ________ . A. a dog B. no dog C. two dogs D. a white dog 58. Ma Jun is ________. A. Wu Dong and Wang Hong’s friend. B. Wu Dong’s friend. C. Wang Hong’s friend. D. Li Ming’s friend. 59. Where is Ma Jun’ s dog? A. In the room. B. Under the tree. C. On the sofa. D. We don’t know. 60. Which of the following is right according to this dialogue? 根据对话,哪一项是正确的? A. The four boys have no dogs. B. Wang Hong’s dog is lost. C. Ma Jun’ s dog is yellow. D. Ma Jun has many dogs. B Everyone 人人needs to be healthy. Do you eat healthy food every day? It’s important 重要for your health. You need fruit, like oranges, apples and bananas. “ An apple a day can keep away避开 the doctor 医生.” You also need vegetables, like carrots and broccoli. Fruit and vegetables are good for our health. But don’t eat lots of chocolate. It’s not good healthy food. What about dessert? Don’t eat lots of dessert because it’s not good for your health. Many people eat lots of chocolate and dessert, so they are unhealthy. Sports can also keep you healthy. Play sports every day. Don’t be lazy. You can be healthy. are healthy food? A. Fruit and vegetables. B. Bananas, apples and chocolate. C. Fruit and dessert. D. Vegetables and dessert. keeps you healthy? A. Food. B. Sports. C. Healthy food. D. Healthy food and sports. 63. 文中划线句子“ An apple a day can keep away the doctor”的意思是________. A. The doctor doesn’t like an apple. B. You don’t need a doctor. C. You bring an apple to the doctor and he runs away跑走. D. You eat an apple a day and you can be healthy. 64. 猜测文中划线单词unhealthy的意思:________. A.健康 B.不健康 C. 饥饿 D. 不饿 65. Which is Right? 哪一项是正确的? A. Everyone is healthy. B. We need to be healthy. C. Food isn’t important. D. A doctor is important. C I have a good friend. Her name is Linda. She has a red backpack. It is on the sofa. She has two blue dictionaries. They are on the desk in her room. She has a yellow quilt. It is on her bed. She likes tennis. She has four tennis rackets and eight tennis. They are under her bed. She likes volleyball, too. But she doesn't like soccer. She thinks认为 it is for boys. She plays tennis with her sister in the morning and plays volleyball with me in the afternoon. color is Linda's backpack? A. Red B. Black C. Blue D. Yellow are Linda's dictionaries? A. On the sofa B. On the desk C. On the bed D. Under the bed has tennis. A. two B. four C. five D. eight plays tennis . A. in the morning B. in the afternoon C. with her friend D. with her brother 70.下面哪项陈述是错误的? A. Linda's dictionaries are blue. B. Lind's quilt is on her bed. C. Linda doesn't like volleyball. D. Linda thinks soccer is for boys. D Found: Is this your book? Please call John 495-3456. Lost: My school ID card. My name is Mike. Please call 487-2349 Found: Is that your black backpack? Please call Mary. Phone#476-5939 Found: A set of keys. Please call Lily 498-2456 Lost: My pencil case. Blue and white, call Tom 456-8700. Found: A cat. Black and White. At the school gate. Call Lisa 412-9856. found发现了 . A. a blue and white cat B. a black and white pencil case C. a black backpack D. keys 如果 you find a school ID card ,it may be可能是 。 's B. Lisa' C. Tom' s D. John's found a cat . A. in the school B. at home C. at the school gate D. in the classroom 74. If you lost your keys, you may可以 call to find it . A. 487-2349 B. 476-5939 C. 412-9856 D. 498-2456 can see notices公告in the LOST AND FOUND. C. 2 D. 6 第Ⅱ卷非选择题 共40分 V. 词汇 15分 一 根据句意和所给汉语提示在横线上写出英语单词。10分 76. I have an ___________ and milk for breakfast. 鸡蛋 77. I don’t like ___________ . I like English. 数学 78. Tom has ___________ baseballs. 三个 79. Her mother’s keys are in the ___________. 抽屉 is my___________. His name is Alan. 堂兄 81. This is an ___________ game.有趣的. 82. My pen is ___________. 红色的 83. How do you ___________ boy? 拼写 take some apples to ___________.他们 85. Chinese is very ___________ for the twins. 困难的 二根据句意,用方框中所给单词的适当形式填空,每词用一次。5分 has tomato play friend watch 86. Nancy ___________ TV every evening. 87. These ___________are red. 88. Those are my___________ . 89. Let’s ___________basketball. 90. -Does your mother___________ a puter ? - Yes, she does. VI. 补全对话 5分 阅读对话,从每题A、B、C、D四个选项中,选出一个最佳答案完成对话。 A: Bob, ___91__ B: Yes, I do. A: ___92___ B: They are on the bookcase. A: Let’s play puter games. B: ___93___Let’s play soccer! A: OK. Do you have a soccer ball? B: ___94___ But I don’t know where it is. A: look, ___95___ B: Oh, it’s under my bed. 91. A . Do you have puter games? B. Do you have a video tape? C. Do you play sports? D. Do you like ice cream? 92. A. Who are they? B. What are they? C. Where’s it? D. Where’re they? 93. A. I like puter games. B. But I don’t like puter games. C. That sounds relaxing. D. It’s fun. 94. A. Yes, I do. B. No, I don’t. C. Yes, I am. D. No, you don’t. 95. A. is it under your table? B. it’s under your bed. C. Where is it? D. What’s this? VII. 任务型阅读 5分 阅读下面短文,然后根据要求完成短文后面的题目。 I have a friend. His name is Jack. He is healthy because因为 he plays sports. He runs every day. He runs to his school in the morning and he runs home after school in the afternoon. He can run fast. He likes playing soccer, volleyball, tennis, basketball , ping-pong and more. He has three soccer balls, two volleyballs, six tennis rackets, four basketballs and many ping-pong balls. They are in his room. He plays sports with his friends every day. He likes many sports stars and he often watches them on TV. 96. 根据短文内容完成句子每空限填一个单词,1分。 Jack is a ___________, he runs every day and he can run fast. 97. 划线单词them指___________ 分 98. 根据短文内容回答问题。1分 Where are Jack’s volleyballs? _______________________________________________________________________ 99. 把短文中划线的句子译成汉语。2分 ________________________________________________________________________ VIII.书面表达。15分 根据下表Sally的有关资讯, 以”My friend----Sally”为题写一篇短文不少于50个单词。 Name Sally Phone 698-8968 Family Two sisters and a brother Sports Tennis, soccer Food Bananas, chicken and hamburgers Subjects科目 Math, English My friend----Sally 七年级英语期中检测试题答案 1-5 BCACC 6-10 DACBF 11-15 CBCBA 16-20 ABCAB 21-25 BCCBC 26-30 BDCDC 31-35 DAABB 36-40 DCBAB 41-45 DCDDB 46-50 ACDBD 51-55 BABCD 56-60 DBCBC 61-65 ADDBB 66-70 ABDAC 71-75CACDD 76. egg 77. math 78 . three 83. spell 86. watches 87. tomatoes 88. friends 89. play 90. have 91-95 ADBAB 96. runner 97. sports stars 98. In his room. 99. 他健康因为他运动。 VIII.书面表达 I have a good friend. Her name is Sally .Her phone is 698-8968. Her favorite food is bananas, chicken and hamburgers. She likes math and English .She likes tennis . She has four tennis rackets and eight tennis. They are under her bed. She likes soccer, too. But she doesn't like volleyball. She has two sisters and a brother in her family. She plays tennis with her sisters in the morning and plays soccer with her brother in the afternoon.

少年读书,如隙中窥月;中年读书,如庭中望月;老年读书,如台上玩月。皆以阅历之深浅,为所得之深浅耳。下面给大家带来一些关于人教版 七年级英语 期中试卷及参考答案,希望对大家有所帮助。 人教版七年级英语下册期中考试 分数:100分时间:90分钟 一、单项选择题 (15分) ( )1. Everyone ______him, because he is friendly. A. like B. likes C. to like D. liking ( )2. --What ________he do? --He __________ an actor. A. is, is B. does, does C. does, is D. is, does ( )3. Listen! Dave _____________on the phone. A. talks B. are talking C. talk D. is talking ( ) is ugly lion under the tree . A. a B. an C. the D. / ( )5. Look! The man ________ under the tree and enjoying the cool. A. lie B. lying C. is lying D. lies ( )6. --Excuse me, where's the supermarket ? --It is _______ the post office and the library. A. next B. across C. between D. in front of ( ) are you _____? -- My mother. A. wait B. waiting C. waiting for D. wait for ( )8. --Oh, hi, Jane!__________? -- Not bad. A. How it is going B. What are you doing C. How is it going D. What does she do ( ) do animals eat? --Some eat and some eat A. meat; leafs B, meats; leafs C. meat; leaves D. meats; leaves ( ) me, is there a bank over there? --___________. A. Yes, there's B. No, there aren't C. Yes, there is D .No, it isn't ( ) often TV in the evening. But she isn't TV now.. A. watch; watching B. watches, watching C. watches ; watch D. watching; watch ( ) are .They are people's . A. friend; friendly B. friends; friendly C. friendly; friend D. friendly; friends ( )13. What ____ he _____? He is a reporter. A. do, do B. is, does C. is, doing D. does do ( )'s the pandas first . A. see B. to see C. look D. to look ( ) animals do you like ? A. some B. any C. other D. the 二、 完形填空 (10分) Mr. Smith 16 from London. Now he is in China. He is 17 .He teaches 18 a middle school. He works very hard. His students like 19 very much. He can 20 a little Chinese . His students often teaches him Chinese on Sundays. Mr. Smith likes playing soccer. He often plays 21 with his students. Mr. Smith 22 a son. His name is Jack. He is a student. He studies in a middle school. He likes 23 TV. It's 7 :00 . now. Mr. Smith 24 a letter to his friend. Jack is 25 a newspaper. His mother is cleaning the room. They're busy. ( ) B. is C. are D. be ( ). a teacher B. an actor C. a reporter D. a doctor ( ). on B. in C. of D. from ( ). he B. him C. she D. her ( ). say B. speak C. talk D. tell ( ). baseball B. soccer C. ping pong D. basketball ( ). has B. have C. there is D. there are ( )23. A. watch B. watches C. watching D. watched ( ). write B. writes C. is writing D. are writing ( ). looking B. seeing C. reading D. watching 三、阅读理解 (30分) A Today Han Mei's classmates are having an art lesson in the park. Now they are playing near the lake. The boy in black is flying a kite. He is Li Lei. The girl in red is reading a book. Who is she? Ha, she's just Han Mei. Lucy and Lily are twins. They look the same(相似的)and they are dancing under a big tree. There is a boat on the lake. Mrs. Green and Mrs. Brown are in the boat. They are their teachers. And now the children are drawing pictures on the grass near the lake. The teachers are teaching how to draw pictures. But what is Joe doing? Oh, he is looking for his pencil case. ( ) 26. Why are the children in the park today? A. They have no classes today. B. They like playing in the park. C. They're having a class. D. Their teachers like boating. ( ) 27. What is Han Mei doing? A. She is drawing a picture. B. She is flying a kite. C. She is dancing with the twins. D. She is reading a book. () many children are mentioned(提到) in the passage? A. Four B. Five C. Six D. Seven ( ) 29. Where are the girls dancing? A. Under the tree B. In a boat C. In the classroom D. Near the lake ( ) 30. Who is Mrs. Green? A. She is their English teacher. B. She is Joe's mother. C. She is their art teacher. D. She is their music teacher. B There are many clubs in our school. Let me tell you what they are. Do you like playing tennis? It's OK if you can't play tennis. Mr. Qing can teach you. Come and join the tennis club! Do you like acting? Do you like to work with other young people ? Do you want to be in the school play? Come and join the acting club. Can you dance? Do you like Latin (拉丁) dance or Chinese traditional (传统的) dance? Join the dance club and dance for the Christmas ( 圣诞节 ) show. Do you like French food? Can you speak French? Do you want to learn French? Join the French club now. ( ) 31. Who can teach you to play tennis? A. Mr. Wang B. Mr. Qing C. Miss Zhang D. Miss Song ( ) 32. If Tony want to act in the school play, what club can he join? A. The tennis club. B. The dance club. C. The acting club. D. The French. club ( ) 33. How many clubs are there in the school? A. Two B. Three C .Four D. Five ( ) 34. What can you learn in the French club? A. English B. French C. Chinese D. Japanese ( ) kind of article is it? A. A story. B. An ad. sports news D. A TV show. C Mike is seven and begins to go to school this week . It's Sunday . His mother Mrs. Smith doesn't go to work. Mike wants to go to the zoo. He gets up at six thirty and asks his mother to take him there. After breakfast they go to the bus stop. They want to take a bus to the zoo . "Look ,Mummy!" the boy is calling out, "A bus is coming !" "No, dear" Mrs. Smith is saying ,"It isn't It's ." "You're wrong, Mummy, "Mike is saying. "My teacher says two and one is three !" ( )36. Mike is a student of Grade . A. One B. Two C. Three D. Four ( )37. Mike is going to the zoo with . A. his father B. his mother C. his sister D. his brother ( )38. Mrs. Smith and her son want to go to the zoo by . A. bus B. bike C. boat D. plane ( )39. Mike thinks , so he is calling out . A. Two and one is three B. a bus is coming C. a No. 3 bus is coming D. a bus is coming ( )40. Who is wrong ? A. The teacher B. Mike C. His mother D. A、B and C D Some children are flying kites near the river. They are very high and Peter can see them in the classroom. After lunch his aunt comes to see his mother and brings a kite to him. It looks like a bird. He wants to fly it but his mother doesn't let him do so. "It's time to go to school," says his mother, "You can fly it after dinner. The boy is unhappy, and says goodbye to his aunt and leaves. Now he is in the classroom but he doesn't listen to Miss Black, She writes something on the blackboard and then turns to the class and finds this. "Name nine things with milk(牛奶) in them, Peter." she says. The boy doesn't hear her and still sits there. A boy in Row 4 taps(拍)his back and he understands it. He stands up and Miss Black asks him the question again. He thinks for some time and then says, "Yes, madam, tea, coffee, cake and... er ... six cows." ( ) is ______ near Peter's school. A. some water B. a river C. a lake D. a park ( ) 42. In the classroom Peter can see ______. A. some children playing in the park B. his aunt buy a kite for him C. some students reading beside the lake D. some children flying kites near the river ( ) 43. Peter's aunt comes to see his mother ____. A. in the morning B. after dinner C. in the evening D. after lunch ( ) 44. Peter's mother tells him to fly the kite______. A. at breakfast B. Quickly C. after dinner D. in school ( ) 45. In class Peter only thinks ______. A. his teacher's words B. what the kite looks like C. how to fly the kite D. how to answer the teacher's question 四、任务型阅读 (5分) Li Feng is from Beijing, China. He studies in a middle school. He works hard at his lessons. He is good at English. He likes speaking English. Li Feng's mother is from Japan. She is a good teacher. She teaches Japanese in a different school. She can speak some Chinese, but doesn't know English. His father works in a factory. He is very busy every day, and his job is difficult but interesting. 阅读短文,回答问题 46. Where does Li Feng come from? ______________________________________________________ 47. What does Li Feng like? ______________________________________________________ 48. Can his mother speak a little English? ____________________________________________________ 49. Where does his father work? ______________________________________________________ 50. Does his father like his work? _______________________________________________________ 五、词汇 (20分) (一)根据句意及首字母提示完成单词。(10分) 51. My sister wants to be a shop a__________ . 52. Jimmy is reading a story book in the l . 53. It's r _________ outside. Take an umbrella(雨伞)with you. 54. I'm very h .I want to eat a hamburger.. 55. Our Chinese teacher lives in an a_______ near our school. 56. Look! Jack is s__________ at the pool. 57. Is there a park in the n ? 58. When your father is sleeping , you must be q . 59. There is a big house with a beautiful g . 60. There are all kinds of a in the zoo . (二)用方框中所给词的适当形式填空。(10分) join , walk , sun , on , take , other , Egyptian , because , year , What , how Thank you for 61 CCTV's Around The World program. Right now,we are in Egypt , 62 in the desert(沙漠). It's a beautiful, 63 day! There are many people here 64 vacation .Some are 65photos of the pyramids . 66 are riding camels(骆驼)The 67 are wearing a kind of scarf on their heads. I guess they wear them 68 it's so hot ! Behind me I can see the city of Cairo. I'm looking at five thousand 69 of history ! 70 an interesting place ! 61. 62. 63. 64. 65. 66. 67. 68. 69. 70. are we going to meet are you doing, Tom is the weather outside? D. Do you have a soccer ball E. Let's go and play soccer ball F. Are you watching TV 六、补全对话 (5分) A: 71 ? B: I'm watching TV. A: The TV show is boring. 72 . B: That sounds great. 73 ? A: Yes, I do. I have a new soccer. B: 74 ? A: It's sunny outside. It's good for sports. B: ___________75____________? A: At the door of the park, OK? B: Great. See you. 72. 七、书面表达 (15分) Tony是我的笔友,他来自澳大利亚,现居住上海。今年14岁。音乐和 足球 是他的最爱。他还喜欢中国食物和熊猫。他想在中国当一名记者。今天天气晴朗,他正在和朋友们在公园玩。 请根据以上的信息,写一篇以My pen pal 为题不少于60字左右的短文(关键词已给出) Australia, Shanghai, music, soccer, Chinese food, pandas, reporter, sunny, playing, park 答案 1-5BCDBC 6-10CCCCC 11-15BDDAC 16-20BABBB 21-25BACCC 26-30CDBAC 31-35 BCCBB 36-40ABACB 41-45BDDCC 46. (He comes from) Beijing, China. 47. (He likes) speaking English. 48. No, she can’t. 49. (He works) in a factory. 50. Yes, he does. 68. because 71-75 BEDCA 七、书面表达 I have a pen pal. His name is Tony. He is is from Australia. Now he lives in Shanghai. Music and soccer are his favorites. He also likes Chinese food and pandas. He wants to be a reporter in China. Today is sunny. He with his friends is playing in the park.七年级英语期中试卷及参考答案相关 文章 : ★ 人教版七年级上册英语期末试卷附答案2017 ★ 初一上册英语完形填空专题复习试题及答案 ★ 初一英语脑筋急转弯汇总及答案大全 ★ 2016年七年级英语下册期末试卷及答案 ★ 七年级上英语Unit2测试题附答案 ★ 七年级英语下册时态复习资料及复习题 ★ 初一上册英语重点短语练习题及答案 ★ 七年级英语下册完形填空测试题及答案 ★ 七年级英语上册期末复习资料 ★ 七年级上英语Unit1测试题附答案

  • 索引序列
  • 初一英语考试卷
  • 初一英语考试试卷
  • 初一英语月考试卷
  • 初一英语期末考试卷
  • 初一英语期中考试卷
  • 返回顶部